You are on page 1of 60

1

Keystone Universe of Education


Class: 12th 9979203664/079-40327045

CH-1 Electrostatics
Session-1.1 (Electric Charges and their Properties)
1. Electric charge [Q or q]:
 The intrinsic property of matter by virtue of which it can show electric effects like electric forces, electric fields or
potentials.
 It is a Scalar quantity.
 SI unit: Coulomb(C), CGS unit: stat coulomb or Electrostatic unit (ESU) (1C= 3x 10 9 ESU)
 Dimensions: [M0L0T 1A 1].
 The property which differentiates between the types of charges is called polarity. Based on the polarity, there
are two types of charges - Positive & Negative
 Like charges repel each other whereas unlike charges attract each other.
When equal and opposite charges are added up, the net charge is zero.
Since they show algebraic addition, Benjamin Franklin termed them as Positive & Negative charges.
Historically,

Positive Charge Charge gained on Glass rod when it is rubbed with silk (GPS)
Negative Charge Charge gained on Plastic rod when it is rubbed with fur (PNF)
2. Electrostatics:
The branch of physics dealing with study of forces, fields & potentials arising due to static charge or charged
configurations at rest is known as electrostatics.
3. Structure of Atom:
 The structure of atoms can be described in terms of three
particles: the negatively charged electron, the positively charged
proton, and the uncharged neutron (Fig 1).
 The protons and neutrons in an atom make up a small, very dense
core called the nucleus, with dimensions of the order of 10-15m.
Surrounding the nucleus are the electrons, extending out to
distances of the order of 10-10m from the nucleus.
 The negatively charged electrons are held within the atom by the
attractive electric forces exerted on them by the positively
charged nucleus. The protons and neutrons are held within stable
atomic nuclei by an attractive force, called the strong nuclear
force that overcomes the electric repulsion of the protons.

4th floor, Sigma excellence, opposite Falguni grah udhyog, Vastrapur, Ahmedabad, 9979203664, 079-40327045
2

4. Quantisation of charge:
 Quantisation of charge means Amount of charge on any object is always an integral multiple of charge of an
electron. This is because electrons can be transferred in integers and not in fractional quantity. Hence
minimum amount of charge can only be equal to electronic charge (e = 1.6×10 -19C).
Q = Ne.
 No of e in 1 C = 6.25 x 10 .
- 18

 Charge on proton = +e and charge on electron= -e.


 This idea was given by Millikan through his famous oil drop experiment.
 Relevance and irrelevance of quantisation:
o At microscopic level, when number of electron involved is of the order of few hundreds (easily countable),
charges are treated as discrete (as an individual). At such scale, quantisation is relevant. Eg. Blades of a
grass.
o At macroscopic level, number of electron involved for charged bodies is way too large to be treated them
as discrete. For example, number of electrons in a charged configuration having charge as small as 1 micro
coulomb charge (10-6 C) or1 µC is 6.25 x1012 , which is so large that individual electron picture (grainy
nature) is lost and charge appears to be continuous.
 Quantisation in physics: Mass is not quantised but examples of quantised quantities are energy, magnetic
moment and angular momentum of electron in atomic orbits.
5. Properties of charge:
i) Amount of electric charge in an isolated system is conserved quantity.
ii) Electric charge is invariant. It doesn’t vary with variation in speed of object.
iii) Total charge on a body is obtained by algebraically adding with proper sign.
iv) Amount of charge on any object is always integral multiple of charge of an electron (Quantised).
6. Process of charging:
Transfer of electrons from one object to another causes charging (Electrification).
Protons are never transferred during charging as they are held by very strong nuclear forces where as loosely
bound valence electrons can be transferred easily when external energy is supplied to atom. Addition of electrons
makes object negatively charged and removal of electrons makes object positively charged. Gain or loss of electrons
is called ionization.
There are three ways of charging:
i. Charging by Conduction:
 We know that heat flow takes place when there is temperature difference between the two bodies.
Heat will flow until the two bodies attain a steady state i.e. both the bodies will have same temperature.
Similarly in the case of charging by conduction, the flow of charges will take place between two objects
as long as there is potential difference between the two bodies.
 When transfer of charges take place by direct contact between two objects having potential difference,
then the process is called as charging by conduction
 The flow of charges will stop when the two objects have the same potential and hence the final charge
on both objects is of same sign.
ii. Charging by Rubbing (or Friction):
 When two surfaces are rubbed against each other, frictional force comes into play. This force is
fundamentally electrical in nature. On rubbing work is done against the force and hence atoms acquire
energy. This energy is used to transfer electrons from one surface to another and thus the two surfaces
become charged. In this process, both the surfaces get equal and opposite charges.

4th floor, Sigma excellence, opposite Falguni grah udhyog, Vastrapur, Ahmedabad, 9979203664, 079-40327045
3

iii. Charging by induction (Electrostatic induction):


To understand charging by induction, we first need to understand polarisation.
 Polarisation:
It is the process of separation of centres of positive and negative charges on two opposite sides (or
faces) of an object.

When an object is brought in vicinity of


charged object (electric field), under its influence it gets polarised i.e. charge same as charged object are
push on away face & charge opposite to charged object are pulled on near face. When the object is
polarised, one type of charges are transferred to the ground by conducting wire and therefore only one
type of charge is left on the body. This process is called electrostatic induction.
7. Discharging by earthing/ grounding:
 Earth has a large capacity to hold charges and it can be considered as a charge sink of infinite capacity at 0V
potential. It can hold any amount of charge without increasing its potential.
 Hence, when a charged object is connected to earth, transfer of electron takes place in such a manner that
object is neutralised. This is known as earthing or grounding.
 It is a safety measure for electrical appliances. Green wire in domestic circuit serves the purpose of earthing
in case of any leakage of charge. Green wire (earth wire) takes the leaked charge to earth and hence save
us from electric shocks. `
8. Conductors and insulators:
 The materials with large number of free charge carriers because of which they allow electricity to pass through
them readily are called conductors.
Examples: Metals, human bodies, earth etc.
 The materials which are devoid of free charge carriers and thus offer high resistance to the passage of
electricity through them are called insulators or dielectric.
Examples: Non-metals like glass, nylon, porcelain, wood etc.

4th floor, Sigma excellence, opposite Falguni grah udhyog, Vastrapur, Ahmedabad, 9979203664, 079-40327045
4

Solved examples
Eg 1: Two charge configurations ‘a’ and ‘b’ are given in fig. 3. Which one of them is correct and why? [1
mark]
Sol: Charge configuration ‘a’ is correct because amount of a charge on a
body can only exist as integral multiple of electric charge. It is not possible
to have continuous charge on the body as given in ‘b’ configuration.
Eg 2: Can a body have a charge of 0.8×10-19C? Justify your answer.
[1 mark]
Sol: We know that charge is quantised. Hence, q=ne.
q 0.8 ×10−19
⟹n= = = 0.5
e 1.6 ×10−19
Ans. Since, n is not an integer, hence body cannot have charge of 0.8×10 -19C.
Eg 3: What happens to mass of body when it is charged? [1 mark]
Sol: When an object is negative charged its mass increases and when an object is positively charged its mass
decreases. Change in mass is equals to mass of electrons transferred during charging.
Eg 4: Two identical metallic spherical shells A and B having charges +4Q and -10Q are kept a certain distance
apart. A third identical uncharged sphere C is first placed in contact with sphere A and then with sphere B, then
spheres A and B are brought in contact and then separated. Find the charge on the spheres A and B.
[Outside Delhi 2011] [3
marks]
Sol: When two identical spheres are brought in contact and moved apart, then both of have them will have
equal charge and equal to average of initial charge on them.
Here, charges on A, B and C are represented as follows:

A B C
Initially +4Q -10Q 0
After A and C are made in 4 Q+0 -10Q 4 Q+0
= 2Q = 2Q
contact 2 2
After B and C are made in 2Q (−10 Q)+ 2Q (−10 Q)+ 2Q
= - 4Q = - 4Q
contact 2 2
After A and B are made in 2Q+(−4 Q) 2Q+(−4 Q) - 4Q
=-Q =-Q
contact 2 2
Ans. Hence, final charge on A = -Q and on B = -Q

4th floor, Sigma excellence, opposite Falguni grah udhyog, Vastrapur, Ahmedabad, 9979203664, 079-40327045
5

Eg 5: How can you charge a metal sphere positively without touching it? [textbook] [2 marks]
Sol: To charge an uncharged metal sphere positively, we will bring a negatively charged rod near one end of the
sphere. Because of this, the sphere will become polarised i.e. all the positive charges will be pulled on near face
w.r.t. to the rod and the negative charges will be pushed away to the far face of the metal sphere w.r.t. the
metal sphere. Negative charges are transferred to the ground (or earthed) by connecting a wire with the ground
to the side where these charges are accumulated. When all the negative charges are transferred, the wire is
disconnected. Hence the sphere will only be left with positive charge. Thus the sphere is positively charged
without touching it.
Steps to charge a metal sphere positively without touching it are given in fig 4.

Eg 6: How can two like charges attract each other? [1 mark]


Sol: If one charge is larger than other, the body with larger charge induces equal and opposite charge on the
nearer end of the body with smaller charge. This opposite induced charge on the smaller charge body exceeds
the small like charge initially present on the body with smaller charge. Hence net force is attractive.
Eg 7: How does a positively charged glass rod attract a neutral piece of paper? [1
mark]
Sol: The +vely charged rod induces –ve charge on the closer end and +ve charge on the farther end of paper.
The rod thus exerts greater attraction than repulsion since –ve charge is closer.

4th floor, Sigma excellence, opposite Falguni grah udhyog, Vastrapur, Ahmedabad, 9979203664, 079-40327045
6

Homework sheet 1.1


Theory
1. Define electric charge. Give its SI unit and dimensional formula? [1 mark]

2. State properties of electric charges. [2 marks]


3. What do you mean by grounding or earthing? [KV] [1 mark]
4. (a) Explain the meaning of the statement ‘electric charge of a body is quantised’. (b) Why can one ignore
quantisation of electric charge when dealing with macroscopic i.e., large scale charges? [Textbook]
[2
marks]
Application Based
5. When a glass rod is rubbed with a silk cloth, charges appear on both. A similar phenomenon is observed with
many other pairs of bodies. Explain how this observation is consistent with the law of conservation of
charge. Ans. Here, the initial and final charge on the total system (glass rod and silk cloth) is same as no
charge in lost in process of transfer of charges. [Textbook] [1 mark]
6. Consider three charged bodies P, Q and R. If P and Q repel each other and P attracts R, what is the nature of
force between Q and R? Ans. Attractive [KV] [1 mark]
-21
7. Is it possible to have a charge of 10 C? Why? [DPS] [1 mark]
8. What is the correct test for electrification of an object? Explain why? [1 mark]
9. The following materials are arranged in electrostatic series in which materials, when rubbed against each
other, develop positive charge if they occur before the other material in the series.
Glass, Flannel, Wool, Silk, Hard Metal, Rubber, Wax, Plastic, Sulphur. What is the charge on silk,
(a) When it is rubbed with glass, Ans. Negative Charge
(b) When it is rubbed with wax. Ans. Positive Charge [2 mark]
10. Raju is performing experiment of induction of neutral object by charged negative rod (Fig 3). While
performing the experiment, Raju observes that, while connecting the polarised body to earth, the body does
not get positively charged (there is no transfer of charge to Earth). State two faults made by Raju in the
experiment.
Ans. The two faults are as follows: (i) when Raju connected polarised body to Earth, he did not keep the
negatively charged rod near to polarised body because of which induction did not take place and hence no
charge is observed. (ii) Now, if he again brings the negatively charged rod near the body while it is
connected to earthing, then also all the induced charge is transferred to earth. Here, the correct method will
be to remove the earthing wire, induce the body using negatively charged rod and then connect the
polarised body to earth while keeping negatively charged rod intact. [2 mark]
Value Based
11. Ram and Shyam went to the trade fair. They were busy in a corner where balloons were sold. A child was
seen troubling his parents and crying for something. On seeing this, Ram went to the child and said that he
would perform a trick with balloons. Ram took two balloons and Shyam helped him to inflate and tie these

4th floor, Sigma excellence, opposite Falguni grah udhyog, Vastrapur, Ahmedabad, 9979203664, 079-40327045
7

balloons. The balloons were rubbed with the sweater Ram was wearing. When taken nearer to a wall, the
balloons got stuck to wall. The child enjoyed this and stopped crying.
a. Give two values of Ram and Shyam.
b. How did the balloons get attracted to wall? Will they repel each other?
Ans. When balloons were rubbed with woolen sweater, it becomes negatively
charged. When taken nearer the wall, positive charges are induced by electrostatic induction on that
part of the wall, so gets attracted. Yes, when the bodies are similar charged they repel.
[3 marks]
12. An elderly woman went alone to Registrar’s office to disburse her property. When she enquired in the office
she was asked to Xerox copy of the document which works under electrostatic induction. The Xerox shop
was far away and across the road. She took help of a passer-by and gets the Xerox done.
a. What values did the passer-by have?
b. How does a neutral body get charged by electrostatic induction? [Sample Paper] [3 marks]
Numerical
13. If a body gives out 10 9 electrons every second. How much time is required to get a total charge of 1C?
Ans. 6.25×109s [DPS] [textbook] [2
marks]
14. A polythene piece rubbed with wool is found to have a negative charge of 3×10 -7 C. Estimate the number of
electrons transferred (from which to which?) Ans. 1.875 ×1012 from wool to polythene
[textbook] [2
marks]

4th floor, Sigma excellence, opposite Falguni grah udhyog, Vastrapur, Ahmedabad, 9979203664, 079-40327045
8

Space for classroom notes

4th floor, Sigma excellence, opposite Falguni grah udhyog, Vastrapur, Ahmedabad, 9979203664, 079-40327045
9

Session 1.2 (Coulomb’s Law)


Read in advance: (i) Vector addition from Ch-4 (Class 11 th)
1. Coulomb’s law:
It states that “electrostatic force between 2 POINT SIZED CHARGES AT REST is directly proportional to product of
their charges and inversely proportional to square of distance between them and direction of this force lies
along the line joining two charges (central force)”.
1
Thus, F ∝ q1q2 & F∝
(inverse square law)
r2
q1q2 ¿
⟹ F∝ 2 ⟹F = k c ∨q1 q2 ∨ 2 ¿.
r r
 Coulombian constant (kc):
 Its value depends on medium intervening two charges.
 Dimensions: [ML 3T -4 A-2].
1
 For air or vacuum, kc = = 9x109 Nm2C -2. Here ε 0 (epsilon zero) is known as permittivity of free
4 π ϵ0
space and ε 0=¿ 8.85 x 10-12 C2N-1m-2 & its dimensional formula is [M-1 L-3T 4A2].
 Another unit of permittivity is Farad/m.
 Permittivity of vacuum (ε 0) is least. Hence, for any other medium value of
permittivity (ε ) is greater than ε 0.
 Graph for Coulomb’s Law:
1
F v/s r graph (Fig 5): Since F ∝ , hence F decreases with increase in r and
r2
hence the graph is a hyperbola.
 One coulomb charge:
One coulomb is defined as that amount of charge that repels an equal and
similar charge with a force of 9×109N when placed in vacuum at a distance of 1m from it.
 Relative permittivity:
 The ratio of permittivity of given medium to permittivity of free space is known as relative permittivity or
ε
dielectric constant (εr or K). Mathematically, εr = .
ε0
 It is a unit-less dimensionless ratio.
 In physical sense, it is the property of absorbing electric effect of given medium. So higher the dielectric
constant of the medium, lesser is the electric effect in that medium.
 For vacuum or air, εr or K = 1, whereas for other mediums, K > 1.

4th floor, Sigma excellence, opposite Falguni grah udhyog, Vastrapur, Ahmedabad, 9979203664, 079-40327045
10

 Relation between Coulomb’s force and dielectric constant:


1 q1 q2
By Coulomb’s law, F = →①
4 π ϵ0 r2
If we replace vacuum with another medium of permittivity ε,
1 q1 q2
F’ = →②
4 πε r 2
On ②/①, we get,
F ' ϵ0
⟹ =
F ϵ
F' 1
⟹ =
F ϵr
F F
⟹ F’ = =
ϵr K
 Numerical note:
¿¿
Sign of charges should be ignored while finding magnitude of Coulomb’s force (F = k c ∨q1 q2 ∨ ) in
r2
numerical. Sign of charges only decides the direction of force. Like charges repel and unlike charges attract.
2. Vector form of Coulomb’s law (Examiner’s Favourite):

r 1 and ⃗
Let us consider two charges q1 & q2 having position vector ⃗ r 2 in X-Y plane. These like charges placed in
F 12 & ⃗
vacuum repel each other with forces ⃗ F 21respectively.
r 21 = ⃗
Here, ⃗ r2 - ⃗
r 1 →① [Displacement Vector = Final position vector – Initial position vector]
r⃗12 = ⃗r1 - ⃗r2 → ②
r 12|=|⃗
|⃗ r 21| = r
r
⃗ r 21

Thus, 12 = r^ 12 & = r^
21
r r
So r^ 12 and r ^21 are the unit vectors which can be used to describe
direction of forces ⃗ F 12 & ⃗
F 21respectively.
By Coulomb’s Law, the forces act along the line joining 2 charges,
k q1 q2
F 21=|⃗
Hence, ⃗ F21|. r^21= . r^
21 [⃗A = A. ^
A]
r2
k q 1 q2
F 12=|⃗
Similarly, ⃗ F12|. r^12= 2 . r^12
r
But we know that, r^ 21 =¿ - r
^12
kq q kq q
Therefore, ⃗F 21 ¿− 12 2 . r^12 & ⃗ F 12 ¿− 12 2 . r^21
r r
F
⃗ ⃗F
Thus we get, 12 ¿ - 21.
kq q r
F 12 = 12 2 r^12.
Also, ⃗
3. Deductions r
r from Vector Form of Coulomb’s Law:
 Force between like charges is repulsive and will bear +ve sign
whereas force between unlike charges is attractive and will
bear -ve sign.

4th floor, Sigma excellence, opposite Falguni grah udhyog, Vastrapur, Ahmedabad, 9979203664, 079-40327045
11

 Coulomb forces occur in action - reaction pair (follow Newton’s 3 rd law of motion).
 Magnitude of force on each charge will be equal irrespective of their charge.
4. Limitations of Coulomb’s law:
 Coulombs law is not valid for moving charges because moving charges produces magnetic field also.
 Coulombs law is valid for point charges only.
 Coulombs law is fully valid only if distance between 2 charges of the order of few Ȧ , if less than this then
nuclear forces come into play which are much stronger than coulombic forces.

5. Principle of superposition:
 According to the principle of superposition, total force acting on
a given charge due to number of charges is the vector sum of
the individual forces acting on that charge due to all the charges.
 Consider number of charges q 2, q3, q4 … are applying force on a
F net = ⃗
charge q1 ; Net force on q1 will be ⃗ F 12 + ⃗
F 13 +…..+ ⃗
F 1n
 Principle of superposition is applicable to Coulomb’s law
because Colombian forces between two charges are
independent of the presence of other charges present around
them. That is why it is called two body force.
6. Properties of Coulombian Forces:
i. Coulombian forces are central forces; they follow inverse square law and are conservative forces.
ii. Electrostatic force is one of the four fundamental forces.
iii. These forces follow Newton’s third law; they occur in action-reaction pairs.
iv. It is a two body force.

4th floor, Sigma excellence, opposite Falguni grah udhyog, Vastrapur, Ahmedabad, 9979203664, 079-40327045
12

Solved examples
Eg 8: What happens to Coulombic force between two point charges when placed in a medium of dielectric
constant K? Hence, explain why water is a great solvent? [1 mark]
F
Sol: The Coulombian force F between two charges becomes when the two charges are brought in a medium
K
of dielectric constant K. Dielectric constant for water is 81; so force between cations and anions reduces to 1/81
times, thus cations and anions get dissociated. That's why water is such great solvent for ionic compounds.
Eg 9: Compare the strength of Coulombic and Gravitational force by determining the ratio of their
magnitudes (i) for an electron and a proton, (ii) for two protons and (iii) for two electrons. [textbook]
[2
marks]
Gm1 m2
Sol: We know that, gravitational force between two bodies is given by, F G = and electrostatic force
r2
k q1 q2
between two bodies is given by, F E = .
r2
Substituting values of mp =1.67×10-27kg, me = 9.11×10-31kg, qp = qe = 1.6×10-19C, we get,
Ratio of gravitational force and electrostatic force between (i) One proton and one electron 10 –39, (ii) Two
protons is 10–36 (iii) Two electrons is 10 –43. Hence, electrostatic force is very strong as compared to gravitational
force.
Eg 10: What is the value of εr for metal? [1 mark]
Sol: Value of εr for metal is infinite (∞ ).

Eg 11: Plot a graph showing the variation of Coulomb force (F) versus ( r1 )
2 where r is the distance between

the two charges of each pair of charges: (1μC, 2μC) and (2μC, – 3μC). Interpret the graphs obtained.
[Compartment 2011, Outside Delhi 2011] [2
marks]

4th floor, Sigma excellence, opposite Falguni grah udhyog, Vastrapur, Ahmedabad, 9979203664, 079-40327045
13

1
Sol: Fig 9(a) and 9(b) are the graphs for F v/s for charges (1μC,
r2
2μC) and (2μC, – 3μC) respectively. The slope of the graph indicates
the value of kq1q2 (greater is the product, greater is the strength of
the force) for constant values of charge q1 and q2.
Since, for charges (2μC, – 3μC) product of charges is greater, hence
its slope is greater.
Note: Repulsive forces are drawn in first quadrant and attractive
forces are drawn in fourth quadrant.
Eg 12: A charge q is placed at the centre of the line joining two equal charge Q separated by ‘a’ distance as
shown in Fig 10. Show that the system of three charges will be in equilibrium if q = — Q/4.
[Outside Delhi 2013] [2
marks]
F net=0 on any one of the charges. Let us consider charge Q on left side.
Sol: For the system to be in equilibrium, ⃗
kqQ
4 kqQ
Let force on Q due to q be F1= a 2 =
( ) a2
2
k Q2
And due to Q be F2 = 2
a
Here, Fnet = F1 + F2
4 kqQ k Q 2
⟹ 0= + 2
a2 a
4 kqQ k Q 2
⟹- = 2
a2 a
⟹ -4q = Q
⟹ q = -Q/4 Hence Proved.

Eg. 13: Two point charges 2μC and -8μC are placed 1m apart from each other. Where should the third charge
-3μC be placed so that net force on -3μC is zero? [2
marks]
Sol: Let the neutral point be x m away from 2μC charge as shown in fig 11.
Net Force on -3μC charge = F-32 + F-3-8 = 0
k (−3 × 10−6 )(2× 10−6 )
⟹ +
x2
k (−3 × 10−6 )(−8 ×10−6)
=0
(1+ x)2
2 8
⟹ 2 = 2
x (1+ x)
⟹ (1+x)2 = 4x2
⟹ 1+x2 +2x = 4x2
⟹ 3x2 -2x -1 = 0
⟹ 3x2 – 3x + x – 1 = 0
⟹ (3x + 1) (x - 1)=0
⟹ x = -1/3m, 1m

4th floor, Sigma excellence, opposite Falguni grah udhyog, Vastrapur, Ahmedabad, 9979203664, 079-40327045
14

Ans. Thus, at distance 1 m away from 2μC charge, net force on -3μC charge is zero. Here, 1/3m is ignored as
neutral point cannot be between opposite charges.
Eg 14: Consider three charges q1, q2, q3 each equal to q at the vertices of an equilateral triangle of side l. What
is the force on a charge Q (with the same sign as q) placed at the centroid of the triangle? [DPS][2 marks]
Sol: In the given equilateral triangle ABC of sides of length l, if we
draw a perpendicular AD to the side BC,

then AD = AC cos 300 =


√3 l and the distance AO of the centroid
2
1
O from A is (2/3) AD = l.
√3
By symmetry AO=BO=CO.
By Coulomb’s law,
k q1 q2 kQq
Force F1 on Q due to charge q at A = = 2
r2 (l/ √ 3)
3 kQq
= along AO
l2
3 kQq
Force F2 on Q due to charge q at B = along BO
l2
3 kQq
Force F3 on Q due to charge q at C = along CO
l2

Here, forces F1, F2 and F3 are equal in magnitude and hence, F1 = F2 = F3 = F


Here, as can be seen from geometry, F2 and F3 are at an angle of 1200 to each other.
Thus, their resultant is given by parallelogram law of vector addition.
Hence, F23 = √ F2 + F 2+ 2 ( F )( F ) cos 120
= F2 + F 2+ 2 ( F )( F ) (−1/2)

= √ F2 + F 2−F 2
F23 = F
3 kQq
Thus, the resultant of forces F2 and F3 is along OA.
l2
Now, F23 and F1 are of same magnitude and opposite in direction.
Hence, Fnet = F – F = 0
Ans. The net force on charge Q at centroid is zero.

Eg 15: Consider the charges q, q, and –q placed at the


vertices of an equilateral triangle ABC respectively. What is
the force on each charge?
[2 marks]
Sol: The forces acting on charge -q at C due to charges q at
B and q at A are F12 along BA and F13 along AC respectively,
as shown in Fig. 13.
k q2
Here, F31 = F32 = F = [By Coulomb’s Law]
l2

4th floor, Sigma excellence, opposite Falguni grah udhyog, Vastrapur, Ahmedabad, 9979203664, 079-40327045
15

Angle between F32 and F31 is 600.


Hence, by parallelogram law, net force at C is,
F3 = √ F2 + F 2+ 2 ( F )( F ) cos 60
= F2 + F 2+ 2 ( F )( F ) (1/2)

= √ F2 + F 2+ F 2

F3 = √ 3F =
√3 k q2
l2
For charge at A and B,
The forces are shown by F13 and F12 for charge at A and by F23 and F21 for charge at B.
k q2
Here, F12 = F13 = F23 = F21 = F = [By Coulomb’s Law]
l2
Now, angle between F12 and F13is 1200 and between F23 and F21 is 1200.
Hence, by parallelogram law, net force at A and B is,
F1 = F2 = √ F2 + F 2+ 2 ( F )( F ) cos 120
= F2 + F 2+ 2 ( F )( F ) (−1/2)

= √ F2 + F 2−F 2
k q2
F1 = F2 = F = 2
l

Ans. Force on charge at C is


√3 k q2 and on charges at A and B is k q2 .
l2 l2
Eg 16: The sum of two point charge is 7µC. They repel each other with a force of 1N when kept 30cm apart in
free space. Calculate the value of each charge? [2 marks]
Sol: Let two point charges have charge q1 and q2.
Hence, q1 + q2 = 7×10-6 →①
k q1 q2
Now, by Coulomb’s law, F = . Hence,
r2
(9 ×10 9)q 1 q 2
⟹1= 2
(0.3)

⟹ q1q2 = 10-11 or 10×10-12


From ①,
⟹ q1(7×10-6 - q1) = 10×10-12
⟹ q12 - 7×10-6q1 + 10×10-12 = 0
On solving, we get,
⟹ q1 = 5×10-6C or 2×10-6C
Hence, q2 = 2×10-6C or 5×10-6C.
Ans. The two charges are 5×10-6C and 2×10-6C.

Eg 17: Four point charges 2µC, -5µC, 2µC and 5µC are placed at the four corner of A, B, C and D of a square
respectively. If each side of square being 10cm, what is the force on a charge 1µC placed at the centre of
square? [2 marks]

4th floor, Sigma excellence, opposite Falguni grah udhyog, Vastrapur, Ahmedabad, 9979203664, 079-40327045
16

Sol: In fig 14, the forces by 2µC, -5µC, 2µC and 5µC on 1 µC are shown
by F21, F-51, F21 and F51.
As can be seen, force due to 2µC charges are equal and opposite.
Hence, they cancel out each other.
Also, force due to -5μC and 5μC are in the same direction.
Hence, they add up.
Distance between 5μC and 1μC charge is (Diagonal)/2.

Hence, r =
√ 2( 0.1) = 0.1 m
2 √2
Thus,
(9 ×10 9)(5 ×10−6 )(1× 10−6 )
Fnet = 2 × 0.1 2
( )
√2
Fnet = 18N
Ans. Net force acting on 1μC charge is 18N.
Eg 18: Two opposite corners of a square of length ‘a’ carry charge Q and the other two opposite corners of
the square carry q charge each. If the resultant force on Q is zero, how are Q and q related?
Sol: Forces acting on charge Q are shown in fig 15.
k q1 q2
By Coulomb’s law, F = .
r2
kqQ k Q2
Hence, FqQ = and F QQ = [Length of diagonal = √ 2a]
a2 2 a2
Here, forces FqQ & FqQ are perpendicular to each other.
kqQ 2 kqQ 2 √ 2 kqQ
Hence, their resultant is given by, F = (

Now, F and FQQ are in same direction.


√ a2
) +( 2 ) =
a a2

Thus, net force on Q is Fnet =


√ 2 kqQ + k Q2
a2 2 a2
But, Fnet = 0
√ 2 kqQ k Q 2
⟹ 2 + 2 =0
a 2a


√ 2 kqQ = −k Q2
a2 2 a2
⟹ Q = -2√ 2q
Ans. Q = -2√ 2q

4th floor, Sigma excellence, opposite Falguni grah udhyog, Vastrapur, Ahmedabad, 9979203664, 079-40327045
17

Eg 19: Consider two identical spheres P and Q with charges q on each repel each other with a force F. A third
sphere R of the same size but uncharged is successively brought in contact with the two spheres. What is the
new force of repulsion between P and Q? [Delhi 2011] [2 marks]
Sol: Here, charges on P, Q and R are represented as follows:

P Q R
Initially +Q +Q 0
After P and R are made Q+ 0 Q Q Q+ 0 Q
= =
in contact 2 2 2 2
After Q and R are made Q (+ Q )+(Q/2) 3Q (+ Q )+(Q/2) 3Q
= =
in contact 2 2 4 2 4
2
kQ
Here, by Coulomb’s law, F = 2 , where r is the distance between P and Q.
r
After contact with R, force on P and Q becomes,
Q 3Q
k( )( ) 3 k Q2 3 F
F’ = 2 4 = = .
2 8 r2 8
r
3F
Ans. The new force of repulsion is .
8
Eg 20: Three point charges of +2µC, -3µC and -3µC are kept at the vertices A, B and C respectively of an
equilateral triangle of side 20 cm. What should be the sign and magnitude of the charge to be placed at the
midpoint (O) of side BC so that the charge at A remains in equilibrium? [3 marks]
Sol: The forces on charges are shown in fig. 16
(9 ×10 9)(3 × 10−6 )(2× 10−6 )
Here, FAB = FBC = F =
(0.2)2
Angle between FAB and FBC is 600.
Hence, their resultant is given by,
FABC = √ F2 + F 2+ 2 ( F )( F ) cos 60
= √ F2 + F 2+ 2 ( F )( F ) (1/2)
= √ F2 + F 2+ F 2
FABC = √ 3F
Let unknown charge at point O be q.
AO
Here, in ΔABO, sin 600 =
AB
Hence, AO = AB sin600 =
√ 3 (20)cm = 10 3cm
2

(9 ×10 9)q (2× 10−6 )
Hence, force FAO = 2
(0.1 × √ 3)
Since A is in equilibrium, hence,
FAO = FABC
(9 ×10 9)q (2× 10−6 ) √ 3 (9 ×109 )(3 ×10−6)(2 ×10−6)
⟹ 2 =
(0.1 × √ 3) (0.2)2

4th floor, Sigma excellence, opposite Falguni grah udhyog, Vastrapur, Ahmedabad, 9979203664, 079-40327045
18

9 × √ 3 ×10−6
⟹q=
4
⟹ q = 3.9μC
Ans. Charge at O is 3.9μC to keep charge at A in equilibrium.

Eg 21: Two equal positive charges, each of 2C interact with a third positive charges of 3 C situated as shown
in figure 17. Find the magnitude and direction of the force experienced by the charge of 3C. [3 marks]
k (2 c )( 3 c)
Sol: Force due to charge at A i.e 2c on charge at B is given by F AB =
r2
k (2 c )(3 c)
Force due to charge at C i.e 2c on charge at B i.e 3c is given by FBC =
r2
Consider AOB and COB
r = √ 32 +4 2=5 m
4 3 3
cos = , sin = , tan=
5 5 4
The net force on charge at B i.e 3c is given by
Fres = √ F2 + F 2+ 2 F cos 2
= √ 2 F 2 +2 F 2 cos 2
= 2 F 2 (1+cos 2)

= √ 2 F 2 2 cos 2
= √ 4 F 2 × cos2
Fres = 2F cos
k (2 c )( 3 c) 4
= 2 
r2 5
9
9 10 2 10 3 10−6 4
−6
= 2 
55 5
−3
Fres = 3.45610 N
To find the direction of the force ,
B sin
we use, tan  =
A +B cos
F sin 2 2sin cos ❑ 2sin cos
tan  = = =
F + F cos 2 1+cos 2 2cos 2
3
 tan  = tan =
4
−1 3
  = tan

= 37 w.r.t either force
4
Ans. The magnitude of net force is 3.456 10−3 N and it acts at 37 w.r.t either force.

Eg 22: Ten Positivity charged particles are kept fixed on the x-axis at point x=10 cm,20 cm, 40 cm,…., 100 cm.
The first particle has a charge 1.010-8 C, the second 810-8 C, third 2710-8 C and so on. The tenth particle has a
charge 10-5 C. Find the magnitude of the electric force acting on a 1C charge placed at the origin.
[3 marks]

4th floor, Sigma excellence, opposite Falguni grah udhyog, Vastrapur, Ahmedabad, 9979203664, 079-40327045
19

Sol: Let Q=1 C be the charge placed at origin and q 1, q 2,..., q 10 be the charges placed at distance r 1, r 2,..., r 10
from the origin where q 1=110−8 C, q 2= 8  10−8 C ,..., q 10= 100  10−8 C and r 1= 10 cm, r 2= 20 cm,..., r 10=
100 cm.
Magnitude of electric force acting on a 1C placed at origin will be given by
Fnet = F1+F2+F3+…F10
K q1 Q K q2 Q K q10 Q
= 2 + 2 +….+
r 1 r 2 r 210
q1 q2 q
= KQ
[ 2
+ 2 +… .+ 10
r1 r2 r 210 ]
1× 10−8 8× 10−8 1000× 10−8
= 9×109×1[ + +…..+ ]
( 10 ×10−2 ) 2 ( 20 ×10−2 ) 2 ( 100× 10−2 ) 2
9 ×109 ×10−8 13 23 103
= [ + + . . . . + ]
( 10−2 ) ❑2 1102 22 102 102 102
9 10
= [1 + 2 +. . . . +10]
10−4 102
1011 n(n+1)
= 9  103  [because, sum of n natural nos. = ]
2 2
3
= 495  10
Fnet= 4.95  105 N
Ans. The magnitude of the electric force acting on 1C charge placed at the origin is 4.95  105 N

Homework sheet 1.2


Theory
1. State superposition principle for Coulomb’s law. State reason why it is applicable to Coulombian forces?
[2
marks]
Derivations
2. State Coulomb’s law. Explain it in vector form with proper diagram. [2 marks]
Application Based
3. What is the dimensional formula of ε 0? [DPS] [1 mark]
4. Using Coulomb’s law, define one unit charge. [1 mark]
5. How does the Coulomb force between two point charges depend upon the dielectric constant of the
intervening medium? [Outside Delhi 2013] [1 mark]
6. In Coulomb’s Law, on what factors the value of electrostatic force constant k depends on? [1 mark]
Graph Based
1
7. Plot a graph showing the variation of Coulomb force (F) versus ( )
r2
where r is the distance between the two

charges of each pair of charges: (1μC, 2μC) and (2μC, – 3μC). Interpret the graphs obtained.
[Compartment 2011, Outside Delhi 2011] [2 marks]
Numerical
8. (a) Two insulated charged copper spheres A and B have their centres separated by a distance of 50 cm.
What is the mutual force of electrostatic repulsion if the charge on each is 6.5 × 10 -7C? Ans. 0.015N

4th floor, Sigma excellence, opposite Falguni grah udhyog, Vastrapur, Ahmedabad, 9979203664, 079-40327045
20

(b) What is the force of repulsion if (i) each sphere charged double the above amount, and the distance
between them is halved; (ii) the two sphere are placed in water?(Dielectric constant of water=80)
[Textbook] Ans. (i) 0.24N, (ii) 0.0001875N [3 marks]
9. Consider three charges q1, q2, q3 each equal to q at the vertices of an equilateral triangle of side l. What is
the force on a charge Q (with the same sign as q) placed at the centroid of the triangle?
[DPS] [Textbook] Ans. Zero [2
marks]
10. Consider the charges q, q, and –q placed at the vertices of an equilateral triangle ABC respectively. What is
√ 3 q2 q2
the force on each charge? [Textbook] Ans. on charge -q, on +q charges [2
4 π ε0 l2 4 π ε0 l2
marks]
11. The sum of two point charge is 7µC. They repel each other with a force of 1N when kept 30cm apart in free
space. Calculate the value of each charge? Ans. 5µC, 2µC. [KV] [2 marks]
12. Four point charges 2µC, -5µC, 2µC and 5µC are placed at the four corner of A, B, C and D of a square
respectively. If each side of square being 10cm. What is the force on a charge 1µC placed at the centre of
square? [Textbook] Ans. 18N [2 marks]
13. Two opposite corners of a square of length ‘a’ carry charge Q and the other two opposite corners of the
square carry q charge each. If the resultant force on Q is zero, how are Q and q related?
Ans. Q = -2√ 2q [2 marks]
14. Consider two identical spheres P and Q with charges q on each repel each other
with a force F. A third sphere R of the same size but uncharged is successively
brought in contact with the two spheres. What is the new force of repulsion
between P and Q? [Delhi 2011] Ans. 3F/8 [2 marks]
15. Three point charges of +2µC, -3µC and -3µC are kept at the vertices A, B and C
respectively of an equilateral triangle of side 20 cm (fig 18). What should be the
sign and magnitude of the charge to be placed at the midpoint (M) of side BC so
that the charge at A remains in equilibrium? Ans. 3.9µC [3 marks]
Space for classroom notes

4th floor, Sigma excellence, opposite Falguni grah udhyog, Vastrapur, Ahmedabad, 9979203664, 079-40327045
21

Space for classroom notes

4th floor, Sigma excellence, opposite Falguni grah udhyog, Vastrapur, Ahmedabad, 9979203664, 079-40327045
22

Session 1.3 (Electric Field)


1. Electric field intensity or Electric field ( ⃗E ) :
 The electric field intensity at any point due to a source charge is defined as the force experienced by a unit
positive test charge placed at that point.
 Electric field due to a charge Q is the space around the charge, in
which any other charge is acted upon by an electrostatic force.
 The electric field due to this charge Q does not exert a force on itself.
 If a charge q0 is brought in the region of electric field of charge Q , then
it will experience electric forces due to electric field of Q.
F

 E=
(IMP)Mathematically, ⃗ , where q0 → 0 (q0 is test charge) so that presence of this test charge does not
q0
affect the source charge Q and its electric field is not changed, therefore expression for electric field intensity
F

E =lim
can be better written as ⃗
q0 →0 q0
 S.I. unit = NC or Vm or JC m-1 and C.G.S. unit – Dyne/stat coulomb.
-1 -1 -1

4th floor, Sigma excellence, opposite Falguni grah udhyog, Vastrapur, Ahmedabad, 9979203664, 079-40327045
23

 Dimensions: [MLT -3A-1]


2. Contact forces v/s Non-Contact forces:
Contact Forces Non-Contact Forces (Action at a distance)
Types of forces that result when the two interacting  Types of forces that result even when the two
objects are perceived to be physically contacting each interacting objects are not in physical contact with
other.  each other, yet are able to exert a push or pull despite
their physical separation.
Eg. Frictional force, Tensional Force Eg. Electrostatic Force, Gravitational Force
3. Concept of Electric field:
When two charges A and B are brought in vicinity of each other at distance ‘r’ apart, then they experience
mutual electrostatic forces but there is small time delay of a few nanoseconds before these forces are perceived
by the other charge even in empty space. Thus the effect of any motion of A on B cannot be instantaneous.
This means that these forces are carried upon by some waves which travel with speed of light to act upon the
other object.
 Therefore, the accelerated motion of charge A produces electromagnetic waves which then propagate
with the speed of light c, reaches charge B and causes a force on B.
Thus we can say that electric field travels in form of electromagnetic waves with the speed of light in a given
medium.

4. Physical significance of electric field:


 Electric field is an elegant way of characterizing the electrical environment of a system of charges.
 Electric field at any point in the space around the system of charges tells us the force a unit positive test
charge would experience if placed at that point (without disturbing the system).
 Electric field is a characteristic of the system of charges and is independent of the test charge.
 Electric field may vary from point to point.
 Each point r⃗ is associated with a unique vector ⃗
E . So electric field is an example of vector field i.e electric
field is vector quantity.
By knowing electric field at any point, we can determine the force on a charged particle at that point. Hence,
F = q0⃗
⃗ E , where q0 is the test charge placed in an electric field ⃗
E.
Electric field plays an intermediary role in the forces between
two charges. Charge ⇌ Electric Field ⇌
Charge
5. Direction of electric field:

4th floor, Sigma excellence, opposite Falguni grah udhyog, Vastrapur, Ahmedabad, 9979203664, 079-40327045
24

It is defined as direction of force experienced by unit positive charge placed in electric field.
Electric field due to a positive charge is always away from the charge (Fig 20(a)) and that due to a negative
charge is always towards the charge (Fig 20(b)).

Direction of electric force on point charge in uniform electric field: In an electric field
⃗E a charge (Q)
F =Q⃗
experiences a force ⃗ E.
 If charge is positive then force is directed in the direction of
field while if charge is negative, force acts on it in the opposite
direction of field (Fig 21).
6. Electric field due to point charge:
 Consider a point charge q placed at origin. To find electric field
at a distance r, we imagine a test charge q 0 placed at that
point.
kq q 0
F=
According to Coulomb’s law, force on charge q 0 is, ⃗ r^
r2
.
F kq

E=
Electric field at that point is, ⃗ = r^ .
q0 r 2
1
This means that E ∝ .
r2
 E.
Thus, at all equidistant points from the charge have the same magnitude of electric field ⃗
 Therefore, electric field due to a point charge is spherically symmetric.
 Electric force between two point charges is symmetric about the line joining the two charges.
 Factors affecting Electric field due to point charge:
a. Source charge, q (not test charge q0)
b. Distance of point from source charge, r
c. Electrical Permittivity of given medium, ε.

7. Principle of superposition:
The resultant electric field at any point due to various charges is
equal to the vector sum of electric fields at that point due to
indicidual charges.
Enet = ⃗
Hence, ⃗ E1 + ⃗
E2 + ⃗
E3 + …. + ⃗
En

1 q1 q2 q3 qn
Enet =

[
4 π ε 0 r 21 P
r
^ 1 P +
r 22 P
r
^ 2 P +
r 23 P
r
^ 3 P + …+
r 2nP
r^
nP
]
8. Neutral points:
Point where net force due to all charges is zero is known as neutral point.
In case of two charges, neutral point lies closer to smaller charge (for like charges, between the two charge; for
unlike charges, outside the two charges.)

4th floor, Sigma excellence, opposite Falguni grah udhyog, Vastrapur, Ahmedabad, 9979203664, 079-40327045
25

Solved Examples
Eg 23: How many electrons should be removed from a coin of mass 1.6g, so that it may just float in an electric
field of intensity 109N/C, directed upward? [2 marks]
Sol: Let n be the number of electrons removed from the coin. Then, charge on coin, q=+ne.
When the coin just floats, Upward force of electric field = Weight of coin
⟹qE or neE = mg
−3
mg ( 1.6 ×10 ) (9.8)
⟹n= = = 9.8×107
eE ( 1.6 ×10−19 ) ( 109 )
Ans. 9.8×107 electrons are to be removed from coin to keep floating in electric field.
Eg 24: Deduce the expression for the electric field ⃗ E due to a system of charges q1 and q2 with position vector
r⃗ 1 and r⃗ 2 at a point r⃗ with respect to the common origin O. [Delhi 2010] [2 marks]
Sol: Consider two point charges q1 and q2 at position vectors ⃗ r 1 and ⃗
r 2 w.r.t. the origin O. We have to determine
the electric field at point P whose position vector is r⃗ .
Here, r^
1 P and r^2 P are the unit vectors from q 1 to P and q2 to P

respectively.
k q1 k q2
E1 =
Hence, ⃗ 2
r^
1 P and E2 =
⃗ r^
2 P,
r 1P r 22 P
where r1P and r2P are the distances of q1 and q2 from point P.
Thus, by Superposition Law, net electric field at point P is,
E=⃗
⃗ E1 + ⃗E2
k q1 k q2
= 2
r^
1P + r^
2P
r 1P r 22 P

Eg 25: ABCD is a square of side 5m. Charges of 50C, -50C & 50C are placed at A, C & D respectively. Find the
resultant electric field at B. [3 marks]
Sol: Electric fields due to charges at vertices A, C and D is as shown
in fig 25.
kq ( 9 ×109 ) (50)
We know that, E = 2 . Hence, E A = E C = 2
= 18×109N/C
r 5
( 9 ×109 ) (50)
Similarly, ED = 2 = 9×109N/C
(5 √2)
Enet at B = ⃗
⃗ EA + ⃗
EC + ⃗
ED

4th floor, Sigma excellence, opposite Falguni grah udhyog, Vastrapur, Ahmedabad, 9979203664, 079-40327045
26

Now, we identify the pairs of electric fields which are perpendicular or parallel (or antiparallel) to each other.
Here, there are no parallel (or antiparallel), electric fields but E A & EC are perpendicular to each other.
Since EA & EC act in perpendicular directions, hence, resultant of E A & EC is given by,
EAC = √ (18 ×10 9)2 +(18 × 109)2 = 18√ 2×109N/C
Now, EAC & ED are perpendicular to each other. Hence, their resultant is given by,
9 2 9 2
Enet = √(18 √ 2× 10 ) +(9 ×10 ) = 27×10 N/C 9

Ans. Enet = 27×109N/C


Eg 26: Consider a particle of mass m and charge (–q) enters the region between the two charged plates
initially moving along x-axis with speed vx. The length of plate is L and an uniform electric field E is maintained
between the plates. Show that the vertical deflection of the particle at the far edge of the plate is qEL2/(2m vx2).
[3
marks]
Sol: Since the particle is –vely charged, hence it is deflected towards +ve plate or opposite direction of electric
field. The particle moves along a projectile motion, so we consider horizontal and vertical motions separately.
For Horizontal direction, u=vx, a=0, s = L and v=vx.
L
Applying 2nd eqn of motion (s = ut + ½ at2), we get, t =
vx
qE
For Vertical direction, u=0. Here, F= qE ⟹ ma= qE ⟹a =
m
Using 1st eqn of motion (v=u+at), where t is same as in horizontal
qE L qEL
direction. Hence, vy = . =
m v x m vx
Now, using 3rd eqn of motion (v2 = u2 + 2as), we get,
qEL 2 qE
( ) ( )
m vx
=2
m
sy

qE L2 q
Ans. sy = 2 . Thus, Sy or vertical deflection ∝
2m v x m
Eg 27: Fig. 27 shows tracks of three charged particles in a uniform electrostatic field. Give the signs of the
three charges. Which particle has the highest charge to mass ratio? [Textbook] [1
mark]
Sol: The particle which deflects the most has the
highest charge to mass ratio (from eg 17). Here
particle ③ has highest deflection, hence has
highest charge to mass ratio.
Also, particles ①&② have –ve charges as they
are deflected towards +ve plate and charge ③ is
+vely charges as it is deflected towards –ve plate.
Eg 28: Suppose that the particle in eg 26 is an electron projected with velocity vx = 2.0 × 106 m s-1. If E between
the plates separated by 0.5 cm is 9.1 × 10 2 N/C, where will the electron strike the lower plate? [2 marks]
Sol: Here, vertical displacement, sy = 0.5cm as it is the distance travelled by charged particle against the electric
field.

4th floor, Sigma excellence, opposite Falguni grah udhyog, Vastrapur, Ahmedabad, 9979203664, 079-40327045
27

qE
Also, we know that a = . Here, charge is electron, thus, q=1.6×10 -19C and m=9.1×10-31kg. Thus, a =
m
(1.6 ×10−19)(9.1 ×102 )
= 1.6 × 1014ms-2
9.1 ×10−31
2s 2(5 × 10−3 )
Now, using s=ut + ½ at2 for vertical direction (u=0), thus, t =
√ √
a
=
1.6 ×10
Hence, horizontal displacement, s = v xt = (2.0 × 10 )( 8×10 ) =16×10-3m = 1.6cm
6 -9
14
= 7.9×10-9s≅ 8×10-9s

Ans. 1.6 cm
Eg 29: Two point charges 5×10-19C and 20×10-19C are separated by a distance of 2m. Find the point on the line
joining between them at which electric field intensity is zero. [2 marks]
-19
Sol: Let us assume the point to be at x m from 5×10 C.
Hence, the point will be at (2-x)m from 20×10 -19C.
Since both are positive charges, hence, their electric fields at the point will be in opposite directions.
Thus, E5 = E20
( 9 ×109 ) ( 5 ×10−19 ) ( 9 ×109 ) ( 20 ×10−19 )
⟹ =
x2 (2−x)2
1 4
⟹ 2= 2
x (2− x)
1 2
⟹ =
x 2−x
⟹ 2 – x = 2x
⟹ 3x = 2
2
⟹x= m
3
Ans. At 2/3 m from 5×10-19C, electric field will be zero.
Eg 30: Two point charges q1=0.2C and q2=0.4C are placed 0.1m apart. Calculate the electric field at
(a) the midpoint between the charges, (b) a point on the line joining
q1 and q2 such that it is 0.05m away from q2 and 0.15m away from q1.
[3 marks]
Sol: (a) Directions of electric fields at midpoint are shown in fig 28.
Here, r = 0.05m for both the charges q 1 and q2.
Since, E1 and E2 are in opposite directions, hence, net electric
field is given by,
Enet = E2 – E1
( 9 ×109 ) (0.4) −( 9 ×10 9 ) (0.2)
=
(0.05)2 (0.05)2
( 9 ×109 ) (0.2)
=
(0.05)2
Ans. Enet = 7.2×1011N/C from q2 to q1.
(b) Directions of electric fields at midpoint are shown in fig 26.
Here, r = 0.15m for charge q1 and r=0.05m for charge q2.
Since, E1 and E2 are in same direction, hence, net electric field is given by,
Enet = E2 + E1

4th floor, Sigma excellence, opposite Falguni grah udhyog, Vastrapur, Ahmedabad, 9979203664, 079-40327045
28

( 9 ×109 ) (0.4) + ( 9 ×10 9) (0.2)


=
(0.05)2 ( 0.15)2
Ans. Enet = 1.52×1012N/C from q1 to q2.

Eg 31: Four charges +q, +q, -q, -q are placed respectively at the four corners A, B, C and D of a square of side
‘a’. Calculate the electric field at the centre of the square. [2 marks]
Sol: Fig. 29 shows direction of electric field for four charges at center of the square.
As can be seen, Electric fields of charges at A and D are acting in the same
direction and hence will add up.
Similarly, for charges at B and C, electric fields will add up.
a
Here, r = Diagonal/2 = for all the charges.
√2
kq
2 kq
Also, |E+q| = |E -q| = E = a 2 = 2
( ) a
√2
2 kq 2 kq 4 kq
Now, EBC = EAD = E+q + E – q = 2 + 2 = 2
a a a
EAD and EBC act perpendicular to each other.
4 kq 2 4 kq 2 4 √2 kq
Hence, their net electric field is given by, E net = E2AD + E2BC = (

4 √2 kq
√ a2
) +( 2 ) =
a a2

Ans. Net electric field at the center of the square is


a2
Eg 32: An electron falls through a distance of 1.5 cm in
a uniform electric field of magnitude 2.0 × 10 4 NC-1 [Fig
30]. The direction of the field is reversed keeping its
magnitude unchanged and a proton falls through the
same distance. Compute the time of fall in each case.
[3 marks]
Sol: Here, vertical displacement = 1.5cm = 0.015m and E =
2.0×104NC-1
Force acting = qE
By Newton’s second law, F = ma.
Hence, ma = qE
qE
⟹a=
m
Applying, 2nd equation of motion, s = ut + ½at2, we get,
⟹s = 0 + ½ at2
2s 2 ms
⟹t=
√ √
a
=
qE
i. For electron, q = 1.6×10-19C, m = 9.1×10-31kg
2 ( 0.015 ) (9.1×10−31)
Hence, t =
√ −19
( 1.6 × 10 ) (2 ×10 )4
= 2.9×10-9s.

ii. For proton, q = 1.6×10-19C, m = 1.67×10-27kg

4th floor, Sigma excellence, opposite Falguni grah udhyog, Vastrapur, Ahmedabad, 9979203664, 079-40327045
29

2 ( 0.015 ) (1.67 × 10−27)


Hence, t =
√ −19
( 1.6× 10 ) (2 ×10 ) 4
= 1.3×10-7s.

Ans. Time of fall for, (a) electron = 2.9×10 -9s, (b) proton = 1.3×10-7s

Homework sheet 1.3


Theory
1. Define electric field. Give its SI unit. What is the physical significance of electric field in electrostatics?
[Outside Delhi 2013]
[2 marks]
F
2. The electric field E due to a point charge at any point near it is defined as E= lim , where q is the test
q →0 q
charge and F is the force acting on it. What is the physical significance of lim
q →0
¿ in this expression?
[Prakash, MAV, Udgam] [1
mark]
3. What is the nature of symmetry of field due to point charge? [DPS] [1 mark]
Derivations
4. Deduce the expression for the electric field ⃗ E due to a system of charges q1 and q2 with position vector r⃗ 1
and r⃗ 2 at a point r⃗ with respect to the common origin O. [Delhi 2010] [2 marks]
Application Based
5. Two charges +Q and –Q are kept at (-x2, 0) and (x1, 0) respectively in an x-y
plane. Find the magnitude and direction of net electric field at the origin
(0, 0). [Outside Delhi 2009]
1 1
Ans. kQ[ + ] from +Q to –Q [1
x21 x22
mark]
6. A small metal ball is suspended in a uniform electric field with the help of an insulated thread. If high energy
X-ray beams falls on the ball, in which direction will it be deflected? Ans. When X-ray beams fall on ball, it
removes the entire –ve charge from it. Thus, it is only +vely charged and hence is deflected in direction of
electric field. [1 mark]
7. Two point charges q 1and q 2 are placed as shown. The electric field intensity is zero at P. Write two
conclusion that you can draw from this.
[1
mark]

Numerical

4th floor, Sigma excellence, opposite Falguni grah udhyog, Vastrapur, Ahmedabad, 9979203664, 079-40327045
30

8. A charged oil drop is suspended in uniform field 3 ×104Vm-1 so that it neither falls nor rises. What is the
charge on the drop? (mass of the charge 9.9×10-15Kg and g=10 m/s2
Ans. 3.3×10-18C [DPS] [2
marks]
9. Two point charges q C and 4q C are separated by a distance of 2m. Find the point on the line joining
between them at which electric field intensity is zero. Ans. 2/3m from qC [2 marks]
10. Two point charges q1=0.2C and q2=0.4C are placed 0.1m apart. Calculate the electric field at
(a) the midpoint between the charges, (b) a point on the line joining q 1 and q2 such that it is 0.05m away
from q2 and 0.15m away from q1. Ans. 7.2×1011N/C from B to A, 1.52×1012N/C from A to B. [3 marks]
11. Four charges +q, +q, -q, -q are placed respectively at the four corners A, B, C and D of a square of side ‘a’.
kq
Calculate the electric field at the centre of the square. Ans. 4√ 2 [2
a2
marks]
12. An electron falls through a distance of 1.5 cm in a uniform electric field of magnitude 2.0 × 10 4 N C-1 [Fig 33].
The direction of the field is reversed keeping its magnitude unchanged and a proton falls through the same
distance. Compute the time of fall in each case. [Textbook] Ans. 2.9×10-9s, 1.3×10-7s[3 marks]

13. Two point charges q A=3C and q B=-3C


are located 20cm apart in vacuum.
i. What is the electric field at the midpoint O of the line AB joining the two charges?
ii. If the negative test charge of magnitude 1.510−9 is placed at this point, what is force experience
by the test charge? Ans. (a) 5.4106 N/C along OB (b) 8.110−3 N, towards B[3 marks]
14. An oil drop of 12 excess electron is held stationary under a constant electric field of 2.55  104 N/C in
millikan’ s oil drop experiment. The density of oil is 1.26 g cm−3 . Estimate the radius of the drop.
4 3
[Hint: mg = qE ⟹ r g = neE, (947)1/3=9.82] Ans. 9.8210-7 [2 marks]
3

4th floor, Sigma excellence, opposite Falguni grah udhyog, Vastrapur, Ahmedabad, 9979203664, 079-40327045
31

Space for classroom notes

4th floor, Sigma excellence, opposite Falguni grah udhyog, Vastrapur, Ahmedabad, 9979203664, 079-40327045
32

Session-1.4 (Electric Field Lines & Electric dipoles)


1. Electric field lines:
 An electric field line (or electric line of force) is defined as the path
along which a small positive charge would tend to move when free to
do so in an electric field and the tangent to which at any point gives
the direction of electric field at that point.
 Electric field lines are imaginary curves in any given electric field and
the tangent to which indicates the direction of net electric field (Fig
35).
 The strength of the electric field at any point is measured as the number
of lines of force crossing a unit area held normal to the lines of force at
that point.
2. Properties of electric field lines (IMP for Boards):
(i) They start from positive point charge and move radially outward.
They end at infinity.
(ii) Field lines produced due to negative charge start from infinity and
are directed towards the negative charge.
(iii) For a pair of positive and negative charged plates , field lines start from
positive charge and end at negative charge (Fig 36).
(iv) Electric field lines do not form closed loop. This is because field lines start at
+ve charge and end at –ve charge but vice-versa does not take place.

4th floor, Sigma excellence, opposite Falguni grah udhyog, Vastrapur, Ahmedabad, 9979203664, 079-40327045
33

(v) Electric field lines are continuous curve but do not form closed loops. They cannot have sudden breaks
because these lines represent electric field. Electric field may vanish to zero as the distance from source
increases but that would never take place abruptly.
(vi) Field lines never intersect as tangent to field line represent electric field at a given point. At the point of
intersection, there would be 2 possible tangents implying 2 fields in
different direction at one point which is not possible (Fig 37).
(vii) The relative closeness of field lines in different regions of space gives the
idea about the relative strengths of the electric field in different regions.
The more closer the field lines are, the more stronger is the electric field.
(viii) Field lines have the tendency of contraction along the length. This explains that opposite charges attract
(Fig 38(b)).
(ix) Field lines exert lateral pressure on each other. This explains that like charges repel each other
(Fig 38(c)).
(x) The field lines originate (from a positive charge) or terminate (at a negative charge) always at right
angles to the surface of the charge.
(xi) The field lines do not pass through the conductor.
(xii) The number of electric field lines (n) per unit cross-sectional area(A) perpendicular to given area is
n
directly proportional to the magnitude of electric field in that region. Mathematically, ∝E
A

3. Electric field lines due to various types of charge configurations:

4. Electric dipole:
 A pair of equal and opposite charges separated by a small distance (2a) is called electric dipole.
 Dipole moment:
o The dipole moment of an electric dipole is defined as the product of either charge and the length of the
electric dipole. It is denoted by ⃗p.
Mathematically, ⃗p = q × 2a⃗

4th floor, Sigma excellence, opposite Falguni grah udhyog, Vastrapur, Ahmedabad, 9979203664, 079-40327045
34

where is 2a is the separation between the charges or the length of the dipole.
o It is a vector quantity.
o Its direction is from negative to positive charge.
o SI unit of dipole moment is Cm. It measures the strength of an electric dipole.
o Dimensions are [L1A1T1]
5. Physical significance of dipole:
In most of the molecules, like CH4 and CO2, the centres of positive and negative charges lies at the same place.
Hence, their dipole moment is zero. However, these molecules develop dipole moment on applying electric
field. But in some molecules centres of positive and negative charges do not coincide because of which they
have a permanent dipole moment. Such molecules are called polar molecules. Eg., H 2O.
The concept of electric dipole is used in (i) study of electric field of an insulator, (ii) study of radiation of energy
from an antenna.

Consider an electric dipole consisting of 2 point charges-q & +q separated by a small distance 2a. We have
E at a point P at a distance r from the centre of dipole O.
to calculate ⃗
Case 1: When P is on the axis of dipole.
E1 & ⃗
Let ⃗ E2 be the two fields acting at point P due to ‘-q’ & ‘+q’ respectively, then:
kq
E1 =
⃗ p) [where ^p is unit vector in the direction of dipole moment]
2 (- ^
(r + a)
kq
E2 =
⃗ (+ ^p).
(r – a)2
Ep = ⃗
So, net ⃗ E1 + ⃗E2
1
6. Electric Field due to Dipole 1
at different points (Must Do - Examiner Favourite):
= kq ( ^p) 2
[

(r−a) 2
(r + a) ]
kq .4 ar
= 2 ( ^p)
(r −a2 )2
2 kr ⃗p
Ep = 2 2 2
⃗ [as ⃗p = |⃗p|. ^p = q.2a ^p ]
(r −a )
If r>>2a, (if dipole is short)
2 k ⃗p 1 1
Ep = 3
⃗ ⟹⃗
Ep ∝ E∝
[unlike point charge where ⃗ ]
r r3 r2

Case 2: When point P is on equatorial line at a distance r from the centre of the dipole.
Net electric field at point P,
E1 + ⃗
E =⃗
⃗ E2 , where ⃗ E1 is field due to –q at P & ⃗
E2 is field due to +q.
kq
Also, E1| = |⃗
|⃗ E2|= →①
r + a2
2

E1∧¿ ⃗
On resolving ⃗ E2 along equatorial line and perpendicular to that line, we see E1 cosθ & E2 cosθ
add up whereas E1 sinθ & E2 sinθ gets cancelled.
E1|cosθ +|⃗
E ∨¿ =|⃗
Hence, | ⃗ E2|cos θ
E1|cosθ
= 2|⃗
2 kq
= cosθ (from ①)
r 2+ a2
4th floor, Sigmaa excellence, opposite Falguni grah udhyog, Vastrapur, Ahmedabad, 9979203664, 079-40327045
Now, cosθ = 2 2 (from right triangle in fig)
35

7. Important points:
E axial = 2⃗
i. ⃗ E equatorial and both act in opposite directions.
ii. E axial is parallel to ⃗p whereas ⃗
⃗ E equatorial is antiparallel to ⃗p .
iii. The angle between the electric field at an axial point and electric field at an equatorial point is 180°.
JEE focus:
k ⃗p
E due to dipole on any point at an angle θ =
iv. ⃗ . √ 3 cos2 θ+1
r3
1
v. Direction of field from line joining point O to point P at an angle α where tan α = tan θ
2

8. Comparing electric field of dipole to field due to point charge:


 Though the total charge on an electric dipole is zero, but the electric field due to them is not zero. This is
because opposite charges are separated by some distance, i.e., 2a and hence the electric fields due to them
when added do no cancel out each other.
 However, at distances much larger than dipole distance (r>>2a) the fields due to the opposite charges are
nearly equal and hence cancel out.
1 1
 Electric field due to a dipole decreases faster at large distances (E ∝ 3 ), than due to single charge (E ∝ 2 ).
r r

4th floor, Sigma excellence, opposite Falguni grah udhyog, Vastrapur, Ahmedabad, 9979203664, 079-40327045
36

Homework sheet 1.4


Theory
1. Define electric dipole moment. What is its SI unit? What is the difference between electric field due to
dipole and electric field due to point charge? [Foreign 2012, Compartment 2011, Outside Delhi 2011]
[2
marks]
Derivations
2. Derive electric field due to a dipole at an equatorial point. Hence write the expression for the electric field
due to a small electric dipole at a far off point. [Delhi 2015,Foreign 2013,2012,DPS] [3 marks]
3. (a) Derive an expression for the electric field E due to a dipole of length ‘2a’ at a point distant r from the
centre of the dipole on the axial line.
(b)Draw a graph of E versus r for r>>a. [3 marks]
Diagram Based
4. Draw electric field lines due to (i) point sized positive and point sized negative charge, (ii)
two equal like charges placed near to each other and two unlike charges placed near to
each other. [Foreign Set 2013, Udgam]
[2 marks]
5. Sketch the electric field lines for a uniformly charged hollow cylinder as shown in fig 41.
[Udgam] [1 mark]
Application Based

4th floor, Sigma excellence, opposite Falguni grah udhyog, Vastrapur, Ahmedabad, 9979203664, 079-40327045
37

6. An electrostatic field line cannot be discontinuous. Why? [Outside Delhi 2013]


[1 mark]
7. Why do the electrostatic field lines not form closed loops? [Outside Delhi 2014, KV, MAV] [1 mark]
8. Two electric field line never cross each other. Why? [Outside Delhi 2014][Textbook] [1 mark]
9. What is the angle between the direction of electric field at any axial point and any equatorial point due to a
dipole and also the ratio of their magnitudes, when both points have same r?[DPS] [1 mark]
10. Two point charges q1 and q2 placed at a distance of d apart are such that there is no point where the field
vanishes. What can be concluded about the charges q 1 and q2? Ans. Charge q1 and q2 are of opposite sign
[DPS] [1
mark]
11. The distance of field point on the axis of a small electric dipole is doubled. By what factor will the electric
field due to the dipole change? Ans. Electric field will be 1/8 times the original electric field. [1 mark]
12. Two dipole made from charges +q , -q and +Q , -Q respectively, have equal dipole moments . Give the
a) Ratio of the separations of the two pairs of charges
b) Angle between the dipole axes of these two dipoles. Ans. (a)Q:q, (b) 0 [2 marks]
Numerical
13. A system has two charges qA = 2.5 × 10-7 C and qB = –2.5 × 10-7 C located at points A: (0, 0, –15 cm) and B: (0,
0, +15 cm), respectively. What is the electric dipole moment of the system? [DPS] [Textbook]
Ans. 7.5×10-8Cm [2
marks]
14. Two charges ±10 μC are placed 5.0 mm apart. Determine the electric field at (a) a point P on the axis of the
dipole 15 cm away from its centre O on the side of the positive charge,
and (b) a point Q, 15 cm away from O on a line passing through O and
normal to the axis of the dipole (equatorial point). [Textbook] Ans.
2.6×105NC-1, 1.33×105NC-1 [3 marks]
15. Two point charges q1 and q2, of magnitude +10-8 C and –10-8 C,
respectively, are placed 0.1 m apart. Calculate the magnitude of electric
fields at points A, B and C shown in Fig. 42.[Textbook]
Ans. 7.2×104NC-1,3.2×104NC-1, 9×103NC -1 [3 marks]
Space for classroom notes

4th floor, Sigma excellence, opposite Falguni grah udhyog, Vastrapur, Ahmedabad, 9979203664, 079-40327045
38

Session-1.5 (Torque
Consider duedipole
an electric to dipole, Electric
of charges Flux
+q and –q andand Gauss
of length Law) in an uniform electric field ⃗E
2a placed
makinginanuniform
1. Dipole angle θ with
fieldit.(Net
It has dipoleand
Force moment of magnitude
Torque on dipole)p=q×2a.
(Must Force
Doexerted on charge
- Examiner +q by
Favourite):
electric field E = q E
⃗ ⃗
(along ⃗ E ).
Force exerted on charge –q by field ⃗ E = -q⃗
E (opposite to ⃗
E ).
F Total= +q E - q E =0.
⃗ ⃗ ⃗
Hence the net translating force on a dipole in a uniform electric
Field is zero.
But the two equal and opposite forces act at different points of the dipole.
They form a couple which exerts a torque.
Torque= Perpendicular distance between two forces × Either force
τ = 2asinθ × qE = pE sinθ.
As direction of torque is perpendicular to both ⃗p and ⃗ E , so we can write τ⃗ = ⃗p × ⃗E.
 The two pairs of perpendicular vectors in the above expression are (i) τ⃗ and ⃗p and (ii) τ⃗ and ⃗ E
 Special cases:
I. Torque will be maximum when dipole is held perpendicular to ⃗ E , i.e., τmax=pE (Ө=90°).
II. E i.e. τmin=0
Torque will be minimum when dipole is held parallel or anti-parallel to ⃗
4th floor, Sigma excellence, opposite Falguni grah udhyog, Vastrapur, Ahmedabad, 9979203664, 079-40327045
(Ө=0°)
39

2. Dipole in non uniform field:


In a non-uniform electric field, the forces experienced by charges +q and –q are different as electric field is
different at different positions. Hence a net force ⃗ F acts on the dipole and a net torque acts on dipole which
depends upon location of dipole in non-uniform electric field.
In non-uniform electric field,
i. Force:
 When ⃗p is parallel to ⃗ E , then a net force acts on the dipole in the direction of increasing ⃗E,
 When ⃗p is anti-parallel to ⃗ E , then a net force acts on the dipole in the direction of decreasing ⃗
E.
ii. Torque:
If ⃗p is parallel or anti-parallel to ⃗
E , the net torque on the dipole is zero because the sinθ = 0 in both the
cases.

3. Continuous charge distribution:


 We know that electric charge is quantized and the charge on a body is always an integral multiple of the
electronic charge. But in most of the practical situations, charge on a body is so large that the quantization
of charge can be ignored. In such cases, we can consider the charged body as the continuous charge
distributions instead of discrete charges.
Thus, a system of closely spaced charges is said to form a continuous charge distribution.
 The continuous charge distributions may be one dimensional, two dimensional or three dimensional. On this
basis, continuous charge distributions are categorized as follows:
I. Linear charge distribution: In this distribution charges distributed on line. Linear charge density is
defined as charge per unit length. It is denoted by λ.
charge q dq
λ=
length = or
l dl

Total charge on line, q= ∫ λdl


l

SI unit is Cm-1

4th floor, Sigma excellence, opposite Falguni grah udhyog, Vastrapur, Ahmedabad, 9979203664, 079-40327045
40

Dimensions are [L-1A1T1]


II. Surface charge distribution: in this distribution charges are distributed on the surface. Surface
charge density is defined as charge per unit area. It is denoted by σ.
charge q dq
σ=
area = S or dS

Total charge on area, q = ∫ σdS


S
-2
SI unit is Cm
Dimensions are [L-2A1T1]
III. Volume charge distribution: in this distribution charges are distributed on the volume of the body.
Volume charges density is defined as charges per unit volume. It is denoted by ρ.
charge q dq
ρ=
volume = or
V dV

Total charge on volume, q= ∫ ρdV


V
-3
SI unit is Cm
Dimensions are [L-3A1T1]
4. Area Vector:
 It is the vector passing normally to the plane surface.
 By convention, the direction of area vector associated with an area
element of closed surface is along the outward drawn normal to it.

5. Electric Flux:
 It is defined as scalar product of Electric field and surface area of the given surface.
It is represented by symbol φ.
Mathematically, φ = ⃗ E . ⃗S = ES cosθ,
where θ is the angle between electric field and area vector.
 It is a scalar quantity but a property of vector field.
 Its SI unit is Nm2C-1 or Vm.
 Its dimensions are [ML3A-1T-3].
 Electric flux linked with any surface held inside an electric field is the measure of the total number of electric
field lines passing normally through that surface.
 For non-uniform Electric field, Electric flux dφ through a small area element dS due to an electric field ⃗E at
an angle θ with dS is dφ= E .dS =E dS cosθ.
⃗ ⃗

4th floor, Sigma excellence, opposite Falguni grah udhyog, Vastrapur, Ahmedabad, 9979203664, 079-40327045
41

Total electric flux φ over the whole surface S due to electric field E is φ= ∫ ⃗E . ⃗
dS = E S cosθ =⃗
E . ⃗S.
 Electric flux is
I. Positive when 00<θ<900
II. Zero when θ = 900
III. Negative when 900<θ<1800

6. Gauss’s Theorem (Must Do - Examiner Favourite):


The surface integral of the electric field intensity over any closed hypothetical surface (called Gaussian surface)
in free space is equal to (1/ε0) times the net charge enclosed within the surface.

q
E.⃗
ΦE = ∮ ⃗ dS = , where q is the charge enclosed in the surface.
S ε0

Homework sheet 1.5


Theory
1. Define electric flux. Is it a scalar or vector quantity? Give its Si unit and dimensional formula. A charge q is
enclosed by a spherical surface of radius R. If the radius is reduced to half how would the electric flux
through the surface change. [Outside Delhi 2009, Compartment 2013, DPS, Udgam, Board 2018]
Ans. (last part) There is no change in electric flux as it is independent of area by Gauss’s law. [3 marks]
Derivations
2. Derive the expression for the force and torque on an electric dipole kept in uniform electric field. Identify
two pairs of perpendicular vectors in the expression. Show diagrammatically the orientation of dipole in the
field for which the torque us (i) maximum and (ii) half the maximum value.

4th floor, Sigma excellence, opposite Falguni grah udhyog, Vastrapur, Ahmedabad, 9979203664, 079-40327045
42

[Outside Delhi 2014,2012,DPS, KV] [5


marks]
Application Based
3. Comment on direction of net force on dipole in following cases when dipole moment is aligned with
(i) increasing electric field, (ii) decreasing electric field? [KV] [2 marks]
4. Two charges of magnitude -3Q and +2Q are located at points (a, 0) and (4a, 0) respectively. What is the
electric flux due to these charges through a sphere of radius ’5a’ with its centre at the origin.
−Q
Ans. φ = [Outside Delhi 2013, KV] [1
ε0
mark]
5. What is the electric flux through a cube of side 1cm which encloses an electric dipole?
Ans. φ = 0 [Delhi 2015] [1 mark]
6. An electric dipole is placed at rest in a uniform electric field, and released. How will it move (i) for θ=0 0, (ii)
θ=1800,(iii) for 00<θ<1800? State reason. Ans. (i) & (ii) No translational and rotational motion, (iii) Dipole will
rotate till it is parallel to electric field. [2 marks]
Numerical
7. An electric dipole of length 2 cm is placed with its axis making an angle of 30° with a uniform electric field of
105N/C. if the dipole experience a torque of 10√ 3 Nm. Find the magnitude of the charge on the dipole.
[Delhi 2006] Ans. 0.017C [2
marks]
8. A point charges causes an electric flux of -1.0 × 103 Nm2/C to pass through a spherical Gaussian surface of 10
cm radius centred on the charge.
(i) If the radius of the Gaussian surface were doubled, how much flux would pass through the surface?
(ii) What is the value of the point charge? [Textbook] Ans. No change, -8.85× 10-9C [DPS] [2 marks]
E = i^ + 2 ^j + 3k^ N/C. Calculate the electric flux over
9. The electric field in a certain region of space is given by ⃗
S = 3i^ + 4 ^j m2. [Hint: φ = ⃗
the area given by ⃗ E . ⃗S] [Udgam] Ans. 11 Nm2/C [2 marks]

10. A uniformly charged conducting sphere of 2.4 m diameter has a surface charge density (σ) of 80.0 μC/m2.
(a) Find the charge on the sphere. (b) What is the total electric flux leaving the surface of the sphere?
[Textbook] Ans. 1.45×10-3C, 1.6×108Nm2/C [2 marks]
11. A dipole, with its charges, -4µC and +4µC located at point (1, 0, 4) and (2, -1, 5), is present in a uniform
^ /m. How much torque would the dipole experience in this
E =20 iV
electric field⃗
2 a=(2-1)i^ +(-1-0) ^j +(5-4)k^ = i^ - ^j +k^ . Then, τ =q(⃗
field? [Hint: Here ⃗ 2 a×⃗
E )]
Ans. 80×10-6 ( ^j +k^ )Nm [2 marks]
12. In figure 48 calculate the total flux of the electrostatic field through the sphere S 1 and
S2. The wire AB shown here has a linear charge density ‘λ’, given by λ=kx where x is
the distance measured along the wire from the end A. [2 marks]

13. In a certain region of space, electric field is along the z-direction throughtout. The magnitude of electric field
is, however, not constant but increases uniformly along the positive z-direction at rate of 10−5 N/C per
metre. What are force and torque experience by the system having a total dipole moment equal to 10−7 Cm
in the negative z-direction? Ans. 10−12N along negative z direction, zero Nm [2 marks]

4th floor, Sigma excellence, opposite Falguni grah udhyog, Vastrapur, Ahmedabad, 9979203664, 079-40327045
43

14. (i)Consider a uniform electric field E = 3 × 103î N/C. (a) What is the flux of this field through a square of 10 cm
on a side whose plane is parallel to the yz plane? (b) What is the flux through the same square if the normal
to its plane makes a 60° angle with the x-axis? [Compartment 2013] [Textbook]
Ans. (a) 30Nm2/C (b) 15Nm2/C.
(ii)What is the net flux of the uniform electric field of above question through a cube of side 20 cm oriented
so that its faces are parallel to the coordinate planes?[Textbook] Ans. Zero. [3 marks]
15. Two small identical electrical dipoles AB and CD, each of dipole moment 'p' are
kept at angle of 1200 as shown in the fig 40. What is the resultant dipole
moment of this combination? If this system is subjected to electric field (E)
directed along + X direction, what will be the magnitude and direction of the
torque acting on this? [Compartment, Delhi , Outside Delhi 2011]
Ans. p at angle of 300 from +ve x-axis, pE/2, along -ve z-axis
[3 marks]

Space for classroom notes

4th floor, Sigma excellence, opposite Falguni grah udhyog, Vastrapur, Ahmedabad, 9979203664, 079-40327045
44

Session-1.6 (Properties & Applications of Gauss Law)


1. Significance of Gauss Theorem:

4th floor, Sigma excellence, opposite Falguni grah udhyog, Vastrapur, Ahmedabad, 9979203664, 079-40327045
45

 Gauss’s law is true for any closed surface, no matter


what its shape or size, will remain same.
 The term q in RHS of gauss’s theorem includes the
sum of all charges enclosed by the surface. The
charges may be located anywhere inside the surface.
 In the situation when the surface is so chosen that
some charges are located inside and some are
outside, the electric field is due to all the charges,
both inside and outside S whereas flux is taken only due to charges inside. The term q, however, represents
only the total charge inside S.
 The surface that we choose for the application for Gauss’s theorem is known as Gaussian surface. The point
at which electric field is to be calculated should be on the Gaussian surface.
 (IMP)The Gaussian surface chosen should not pass through any discrete charge. This is because electric field
due to a system of discrete charges is not well defined at the location of any charge. As we approach
charges, r decreases and hence electric field grows larger and when r → 0, E → ∞. Hence, electric field is
not defined. However, the Gaussian surface can pass through a continuous charge distribution.
 Gauss’s law is often useful towards a much easier calculation of the electrostatic field when the system has
some symmetry. This is facilitated by the choice of a suitable Gaussian surface.
 Gauss’s law is based on the inverse square dependence on distance contained in the Coulomb’s law. Any
violation of Gauss’s law will indicate departure from the inverse square law.
 Electric field due a charge near an uncharged conductor plate is shown in figure 51.

Consider an infinitely long charged wire with uniform linear charge density λ.
To find electric field ⃗ E at point P having ‘r’ distance from wire, let us consider a right circular cylinder as the
Gaussian surface of radius r & length l.
Electric field ⃗ E due to infinite long wire is radially outwards only.
Net outward flux through the Gaussian surface is given by,
φnet = φcircular ends + φcurved surface
 For circular ends,
E & Area Vector are perpendicular to each other.

Therefore, ⃗ E.⃗dS = ⃗ E⃗ dS cos900 = 0
So, φcircular ends = 0
 For curved surface area,
E & Area Vector are parallel to each other.

Therefore, ⃗ E.⃗dS = ⃗ E⃗ dS cos00 = ⃗
E⃗dS

q
According to Gauss’s law, ∮ ⃗E . ⃗
dS =
ε
.
S 0
2. Electric
❑ Field due to Infinitely long charged wire (Must Do Examiner Favourite):
Here, ∮ dS = 2πrl (curved surface area)

S

qenclosed= λl.
λl
Therefore, |⃗E ∨¿ 2πrl =
ε0
4th floor, Sigma excellence, opposite Falguni grah udhyog, Vastrapur, Ahmedabad, 9979203664, 079-40327045
46

3. Electric Field due to uniformly charged infinite plane sheet(Must Do - Examiner Favourite):
Consider a thin, infinite plane sheet of charge with uniform charge density .
We wish to calculate its electric field ⃗ E at point P at distance ‘r’ from it.
Direction of ⃗ E is normally outwards to the sheet.
E is equal in magnitude but opposite in direction at P & P’ which are at a distance r from sheet.
Also, ⃗
We choose cylindrical Gaussian surface of cross-sectional area A
& length 2r.
Net outward flux through the Gaussian surface is given by,
φnet = φcircular ends + φcurved surface
 For circular ends,
E & Area Vector are parallel to each other.

Therefore, ⃗ E.⃗ A =⃗ E⃗ A cos00 = EA
So, φcircular ends = 2EA (as there are two circular ends)
 For curved surface area,
E & Area Vector are perpendicular to each other.

Therefore, ⃗ E.⃗dS = ⃗ E⃗ dS cos900 = 0
φcurved surface = 0

q
E.⃗
Now, by Gauss’s Law, φnet = ∮ ⃗ dS =
S ε0
Charge enclosed
Consider two largein plane
Gaussian surface
sheets such,that
qenclosed
they= A.
have surface charge densities σ and –σ.
We need to A
calculate σ
electric field in three different regions.
⟹ 2EA = E ¿
ε 2 ε
0 3: Fields 0due to two sheets are
4.In region
Electric1 and
Field between two plane sheets (Must Do - Examiner Favourite):
E is independentσ of ‘r’.
IMP: σ⃗ 1
⃗E1 = and ⃗E2 = - . Therefore, ⃗EI = ⃗E1+⃗ E2 = (σ - σ)=0
2 ϵ0 2 ϵ0 2 ϵ0
In region 2: Fields due to two sheets are
σ σ 1 σ
E1 =
⃗ and
4th E2Sigma

floor, = . Therefore,
excellence, E II Falguni

opposite =⃗E1+⃗ E 2 = udhyog,
grah (σ +Vastrapur,
σ) =
2 ϵ0 2 ϵ0 2 ϵ0 ϵ 0 Ahmedabad, 9979203664, 079-40327045
47

5. Electric field due to uniformly charged thin spherical shell: (Must Do - Examiner
Favourite)

Consider a thin spherical shell of charge q & of radius R with uniform surface charge density .
E is radial & of equal magnitude for equal distances from centre (due to symmetry) .

If a point P is at a distance ‘r’ from centre O, we choose a concentric sphere of radius r as the Gaussian surface.
Case 1: When pt. P lies outside spherical shell (fig 57), the total charge inside
Gaussian surface is equal to charge on surface of conducting sphere,
i.e., q=4π R2 . Flux through Gaussian surface, φE = E  4πr 2

q
E.⃗
By Gauss’s law , ∮ ⃗ dS =
S ε0
q
E |  4πr 2cos 0°=
|⃗
ϵ0
kq σR 2
E| ¿
Or |⃗ ¿ 2 , it is the same as Electric field due to a point charge q placed
r2 ϵ0 r
at the centre O.
Case 2: When pt. P lies on the spherical shell, the total charge inside Gaussian surface is equal to charge on
surface of conducting sphere , i.e. , q=4π R2 (as r=R). Flux through Gaussian surface, φE = E  4π R2

q
E.⃗
By Gauss’s law ,∮ ⃗ dS =
S ε0
q
E |  4π R2 cos 0°=
|⃗
ϵ0
kq σ
E| =
Or |⃗ =
R 2 ϵ0
Case 3: When pt. P lies inside spherical shell. As it is clear from fig. 58,
charge enclosed by a Gaussian surface is 0 ,i.e.,q=0.
Flux through Gaussian surface φE = E  4πr 2.

q
E.⃗
Applying Gauss’s Theorem,∮ ⃗ dS = ,
S ε0
φE = q/ϵ 0  E  4πr =0 or E=0.
2

Note: Einside=0
kq kq
Eoutside= 2 and Esurface = 2
r R

Solved Examples
Eg 33: A charge ‘q’ is placed at the centre of the cube of side ‘l’. What is the electric flux passing through the
(a)each face of cube? (b) two opposite faces of the cube? [Outside Delhi 2011, Foreign 2010] [1 mark]

4th floor, Sigma excellence, opposite Falguni grah udhyog, Vastrapur, Ahmedabad, 9979203664, 079-40327045
48

q
Ans: According to Gauss law, flux through entire cube is given by: . As cube has 6 faces, hence, flux through
ε0
q q
each face is: . Also, flux through two opposite faces will be given by: .
6 ε0 3 ε0
Eg 34: The electric field components in Fig. 45 are Ex = αx1/2, Ey = Ez = 0, in which α = 800 N/C m1/2. Calculate (a)
the flux through the cube, and (b) the charge within the cube. Assume that
a=0.1 m. [DPS] [3 marks]
Sol: For surface 1 at distance a from origin on x-axis, electric field is given by,
E1 = αa1/2 and for surface 2 at distance 2a on the same axis, electric field is
given by, E2 = α(2a)1/2.
For surface 1, electric flux is given by, φ 1 = E1S cos1800 = -αa1/2(a2)
For surface 2, electric flux is given by, φ 2 = E2S cos00 = α(2a)1/2(a2)
For surfaces parallel to y- & z-axis, φ=0 (∵Ey=Ez=0).
Hence, net electric flux through the cube is given, φ = φ 1 + φ2 = αa2 [(2a)1/2-(a)1/2]
Substituting values of α=800N/Cm1/2and a=0.1m, we get,
Φ = 800(0.1)2[(2*0.1)1/2-(0.1)1/2] = 8(0.31)(√ 2 - 1)
Ans. Φ = 1.04Nm2/C
Eg 35: Consider two hollow concentric spheres, S 1 and S2, enclosing charges 2Q and 4Q respectively. (i) Find
out the ratio of the electric flux through them. (ii) How will the electric flux through the sphere S 1 change if a
medium of dielectric ‘εr’ is introduced in the space inside space S1 in place of air? Deduce the necessary
expression. [Outside Delhi 2014] [2 marks]
Sol: (i)Net charge for surface S1= 2Q
2Q
Hence, flux through S1 =
ε0
For surface S2, 2Q charge is induced from S1. Hence, net charge on S2=2Q+4Q= 6Q
6Q
Hence, flux through S1 =
ε0
2Q
ε0
Thus, ratio of flux through S1 and S2 is = = 1:3
6Q
ε0
(ii) If medium of S1 is replaced by a medium of dielectric constant ε r, hence its permittivity becomes εr ε0.
2Q
Hence, flux of S1 becomes =
εr ε 0
2Q
Ans. Ratio of S1 to S2 = 1:3, Flux through S1 =
εr ε 0
Eg 36: Fig 62 shows two large metal plates P1 and P2, tightly held against each other and
placed between two equal and unlike point charges perpendicular to the line joining them.
(i) What will happen to the plates when they are released?
(ii) Draw the pattern of the electric field lines for the system.[Foreign 2009] [2 marks]
Sol: (i) By electrostatic induction, charge induces on
the plates and opposite nature of charge appears on
surface facing each other. Therefore, they start attracting
towards each other.
(ii)Fig 63 shows electric field lines pattern.

4th floor, Sigma excellence, opposite Falguni grah udhyog, Vastrapur, Ahmedabad, 9979203664, 079-40327045
49

Homework sheet 1.6


Derivations
1. State Gauss’ theorem. Apply this theorem to obtain the expression for the electric field intensity at a point
due to an infinitely large, thin, plane sheet of charge. How is the electric field directed if (i) sheet is positively
charged, (ii) negatively charged? [Outside Delhi 2013, Compartment & Delhi 2012, Tulip] [5 marks]
2. Using Gauss law deduce the expression for the electric field due to a uniformly charged spherical conducting
shell of radius R at a point (i) inside and (ii) outside the shell. Plot a graph showing the variation of electric
field as a function of r>R and r<R (r being the distance from the centre of the shell)
[Outside Delhi 2013 & 2011, Compartment & Delhi 2011, K.V.] [5 marks]
3. Using Gauss’ theorem, deduce an expression for the electric field intensity at any point due to a thin,
infinitely long wire of charge/length. Draw a graph to show the variation of E with perpendicular distance r
from the line of charge. [Foreign 2011, DPS, Prakash 2013, M.A.V, KV , Board 2018]
OR
A thin straight infinitely charged long wire having charge density λ is enclosed by a cylindrical surface of
radius r and length l, its axis coinciding the length of the wire. Find the expression for the electric field
through the surface of the cylinder. [Outside Delhi 2011] [4 marks]
4. Using gauss law, derive an expression for the electric field intensity at any point outside a uniformly charged
thin spherical shell of radius R and charge density σC/m2. Draw the field lines when the charge density of the
sphere is (i) positive (ii) negative. [DPS, KV] [3 marks]

Diagram Based
5. A positive point charge +q is kept in the vicinity of an uncharged conducting plate. Sketch electric field lines
originating from the point on to the surface of the plate. [Foreign 2009] [1 mark]
Application Based
6. A spherical conducting shell of inner radius r 1 and outer radius r2 has a charge ‘Q’. A charge ‘q’ is placed at the
centre of the shell. (a) What is the surface charge density on the (i) inner surface, (ii) outer surface of the
shell? (b) Write the expression for the electric field at a point x > r 2 from the centre of the shell. [Hint: Charge
‘q’ will be induced on outer radius sphere and hence total charge on outer radius sphere is ‘Q+q’.]
−q Q+ q k (q +Q)
[Outside Delhi 2010, MAV] Ans. , , [2
4 πr 21 4 πr 22 x2
marks]
7. Gaussian surface cannot pass through discrete charge but can pass through continuous charge distribution.
Explain. [MAV, KV] [1 mark]
8. Two parallel uniformly charge infinite plane sheets. ‘1’ and ‘2’, have charge densities +σ and -2σ respectively.
Give the magnitude and direction of the net electric field at a point (i) in between the sheets and (ii) outside
3σ −σ
near the sheet ‘1’. Ans. (i) in direction from sheet 1 to 2, (ii) in direction inwards to sheet 1
2 ε0 2 ε0
[Outside Delhi 2015]
[2 marks]

4th floor, Sigma excellence, opposite Falguni grah udhyog, Vastrapur, Ahmedabad, 9979203664, 079-40327045
50

9. (a) A point charge q is at a distance d/2 directly above the centre of a square of side d, as shown in the figure
64. Use Gauss’s law to obtain the expression for the electric flux through the square.
(b) If the point charge is now moved to a distance‘d’ from the centre of the square and the side of the square
is doubled, explain how the electric flux will be affected. [Board 2018]
[3 marks]

Numerical
10. A uniformly charged conducting sphere of 2.8 m diameter has a surface charge density of 100μC/m -2. (a) Find
the charge on the sphere. (b) What is the total electric flux leaving the surface of the sphere?
[Delhi 2008, K.V.] Ans. 2.46×10-3C , 2.78×108Nm2/C [2
marks]

11. A conducting sphere of radius 10cm has an unknown charge. If the electric field 20cm from the centre of the
sphere is 1.5103 N/C and point radially inwards, what is the net charge on the sphere?
Ans. 6.6710−9 C [2
marks]
12. An infinite line charge produces a field of 9104 N/C at a distance of 2cm. Calculate the linear charge density.
Ans. 10C/m [2 marks]
13. A hollow cylindrical box of length 0.5 m and area of cross-section 20
cm2 is placed in a three dimensional coordinate system as shown in
E  = 20xi^ , where E
the fig. 65. The electric field in the region is given by ⃗
is in NC-1 and x is in metres. Find (i) Net flux through the cylinder, (ii)
Charge enclosed in the cylinder. [Delhi 2013]
-2 -1 -12
Ans. 0.02Nm C , 0.177×10 C [3 marks]
14. Careful measurement of the electric field at the surface of a black box indicates that the net outward flux
through the surface of the box is 8.0 × 10 3 Nm2/C. (a) What is the net charge inside the box? (b) If the net
outward flux through the surface of the box were zero, could you conclude that there were no charges inside
the box? Why or Why not? [Textbook] Ans. (a) 7.1×10-8C (b) No, because the net sum of charges is zero.
[2
marks]
15. An electric field is uniform, and in the positive x direction for
positive x, and uniform with the same magnitude but in the
negative x direction for negative x. It is given that E = 200 i N/C
for x > 0 and E = –200 i N/C for x < 0. A right circular cylinder of
length 20 cm and radius 5 cm has its centre at the origin and its
axis along the x-axis so that one face is at x = +10 cm and the
other is at x = –10 cm . (a) What is the net outward flux through
each flat face? (b) What is the flux through the side of the
cylinder? (c) What is the net outward flux through the cylinder?
(d) What is the net charge inside the cylinder? [Udgam]
Ans. (a) 1.57Nm /C (b) Zero (c)3.14Nm /C (d)2.78×10-11C
2 2
[3 marks]

4th floor, Sigma excellence, opposite Falguni grah udhyog, Vastrapur, Ahmedabad, 9979203664, 079-40327045
51

Space for classroom notes

4th floor, Sigma excellence, opposite Falguni grah udhyog, Vastrapur, Ahmedabad, 9979203664, 079-40327045
52

Solved Examples
Eg 37: Two infinite parallel plane have uniform charge density σ 1 and σ2. Determine the electric field at point, (i)
To the left of the sheet, (ii) Between them, (iii) To the right of the sheet. [DPS] [2 marks]
Sol: Figure 67 shows two thin parallel sheets having uniform charge
densities σ1 and σ2.
In region 1: Fields due to two sheets are
Consider an infinite line of charge with uniform charge density λ, as shown in fig 68. We wish to calculate
σ1 σ −1
E1the
⃗ = - electricand E2=at- any2 point
field
⃗ P at a distance
. Therefore, EI = ⃗
⃗ E1+y⃗Efrom
2=
it. (σ1+σ2)
2 ϵ
The charge
0 2 ϵ
on small element
0 2
dx of line charge will be, ϵ 0dq = λdx
In region 2: Fields due to two sheets are kdq k λdx
Theσelectric field atσany point P due to the charge element 1 dq will be, dE = 2 = 2 2
E1= 1 and ⃗
⃗ E2= - 2 . Therefore, ⃗ E II = ⃗
E1+⃗ E2 = (σ1 - σ2) r y +x
2 ϵ
The field
0 2 ϵ
dE has two components:
0 dEx = -dE sinθ 2 ϵ 0

In region 3: Fields due to two sheets and aredEY = dE cosθ


Theσ–ve sign in theσx-component indicates that d ⃗E1 acts in the negative x-direction. Every charge element
E1= 1 and ⃗
⃗ E2= 2 . Therefore, ⃗ E III = ⃗
E1+⃗ E2 = x (σ1 + σ2)
on2the
ϵ 0 right has a 2corresponding
ϵ0 charge element on 2 ϵ 0the left. The x-components of two such charge
Egelements
38: Obtainwill be
theequal and opposite
formula and hence
for the electric field cancel
due toout.longThethinresultant
wire of field E gets

uniform contributions
linear only λ without
charge density
from
using y-components
Gauss’s law. and is given by, [3 marks]
x=+∞
Sol: Electric field of a line charge from Coulomb law:
E = Ey = ∫ d E y = ∫ dE cos θ
x=−∞
x=+∞
k λdx
=2 ∫ cos θ
0 y2 + x2
x=+∞
dx
= 2kλ ∫ cos θ 2 2
0 y +x
x π
From fig 61, tanθ = ⟹ x = y tanθ [For x=+ ∞, θ = and for x=0, θ = 0]
y 2
Differentiating both sides, we get, dx = y sec 2θ dθ
π
θ=
2 2
∴ E = 2kλ ∫ cos θ y y2 (1+
sec θ dθ
tan 2 θ)
θ =0
π
θ=
2
= 2kλ sec 2 θ dθ [sec2θ = 1 + tan2θ]
∫ cos θ
θ =0 y sec 2 θ
π
θ=
4th floor,
2 Sigma excellence, opposite Falguni grah udhyog, Vastrapur, Ahmedabad, 9979203664, 079-40327045
= 2 kλ cos θ dθ
53

Eg 39: An early model for an atom considered it to have a positively charged point nucleus of charge Ze,
surrounded by a uniform density of negative charge up to a radius R. The atom as a whole is neutral. For this
model, what is the electric field at a distance r from the nucleus?
Sol: Charge distribution of this model is given in fig 69.
The total negative charge in uniform spherical shell of radius R is –Ze (as charge of nucleus is +Ze and atom as a
whole is neutral).
−Ze
Negative volume charge density ρ of shell is, ρ = 4
π R3
3
To find electric field at point P at distance r from centre, we consider 2 cases: (i) r>R and (ii) r<R.
For both the cases, we will use Gauss law and Gaussian surface will be a spherical shell of radius r from centre of
nucleus.
Case (i): For r>R, charge enclosed= 0 as atom as a whole is neutral.

qenclosed
Hence, applying Gauss law, φ = ∫ E . ds =
S ε0
⟹ E (4πr2) = 0 [∵ qenclosed = 0]
⟹E=0
Case (ii): For r<R, negative charge enclosed by sphere of radius r will be,
4 3 r3
q=ρ π r = -Ze 3
3 R
Thus, net charge enclosed by sphere of radius r = Charge of nucleus + negative charge
r3 r3
qenclosed = Ze - Ze 3 = Ze
R
1−(R3 )
Thus, applying Gauss law,

qenclosed
φ = ∫ E . ds =
S ε0
r3
Eg 40: Show,
E (4πr2) =
Ze (
1− )
with the3help of a suitable example along with the figure, that the outward flux due to a point
R q
ε 0 with a closed surface, is independent of its size and shape and is given by ε .
charge ‘q’, in vacuum
0
[Outside Delhi 2015] [2 marks]

4th floor, Sigma excellence, opposite Falguni grah udhyog, Vastrapur, Ahmedabad, 9979203664, 079-40327045
54

Sol: Here, for simplicity, let charge q be enclosed in a spherical shell of radius R as shown in fig 63.
1 q
For a point charge, electric field at a point at distance R is, E = .
4 π ε0 R 2
Surface Area of a sphere, S = 4πR2.
E . ⃗S = ES cos00 [Electric field and area vector are in same direction as
Electric Flux, φ = ⃗
seen in fig 70]

⟹φ= ( 4 π1ε . Rq )
0
2 4πR2

q
⟹φ=
ε0
q
Hence, electric flux is equal to and is independent of shape and size.
ε0
NOTE: In the above Eg, closed surface chosen can be cube, cylinder or any other surface of your choice. Here,
we have chosen spherical surface for simplified calculations.

Assignment Sheet
1. Three charges, each equal to q are placed at the three corners of a square of side a. Find the magnitude of
kq
electric field at the fourth corner. Ans. (2√ 2+1) 2 [2 marks]
2a
E = 2xi^ (Fig 71), find the net electric flux through
2. Given electric field in a region is ⃗
the cube and the charge enclosed by it. [Delhi 2015] Ans. Zero, 2a3ε 0
[2 marks]
3. Four point charges of charge -0.04μC, 0.04μC, -0.02μC and 0.02μC are placed at
the corners ABCD of a square respectively of side 2cm. Find the magnitude and direction of the electric field
at the centre of the square. Ans. 9√ 2×105NC-1, parallel to BA[3 marks]
4. An electron moves a distance of 6cm when accelerated from rest by an electric field of strength 2×10 -4N/C.
Calculate the time period of travel. Me=9.11×10-31kg and qe=1.6×10-19C. Ans. 5.85×10-5s [2 marks]
5. 64 drops of radius 0.02m and each carrying a charge of 5μC are combined to form a bigger drop. Find the
ratio of initial and final surface charge densities. Ans. 1:4 [2 marks]
-6
6. An electric dipole consists of two opposite charges of magnitude 2×10 C each and separated by a distance
of 3cm. It is placed in an electric field of 2×10 5NC-1. Determine the maximum torque on the dipole.
Ans. 1.2×10-2Nm [2
marks]
7. A thin spherical shell of radius R carries a charge Q on its surface. A point charge Q/2
is placed at its centre C and other charge +2Q is placed outside the shell at a distance
x from the centre as shown in the figure 72. Find the electric flux through the shell.
Q
Ans. [Delhi 2015] [2 marks]
2 ε0
8. Charges of +5C ,+5C, +10C are placed in air at the corners A ,B and C of an
equilateral triangle ABC, having each side equal to 5 cm. Determine the resultant
force on the charges at A. Ans. 238 N [2 marks]

4th floor, Sigma excellence, opposite Falguni grah udhyog, Vastrapur, Ahmedabad, 9979203664, 079-40327045
55

9. Two identical metallic spheres, having unequal opposite charges are placed at a distance 0.90 m apart in air.
After bringing them in contact with each other, they are placed at the same distance apart. Now the force of
repulsion between them is 0.025N.Calculate the final charge on each of them.
Ans.1.5×10-6C, 1.5×10-6C [2
marks]
10. Ten Positivity charged particles are kept fixed on the x-axis at point x=10 cm,20 cm, 40 cm,…., 100 cm. The
first particle has a charge 1.010-8 C, the second 810-8 C, third 2710-8 C and so on. The tenth particle has a
charge 10-5 C. Find the magnitude of the electric force acting on a 1C charge placed at the origin.
Ans. 4.95×105 N [3
marks]
11. Two equal balls having equal positive charge ‘q’ coulombs are suspended by two insulating strings of equal
length. What would be the effect on the force when a plastic sheet is inserted between the two?
Ans. Force decreases by a factor of dielectric constant of plastic sheet [Outside Delhi 2014] [1
mark]
12. The force of attraction between two point charges placed at a distance ‘d’ from each other in a given
medium is ‘F’. What distance apart should they be placed in the same medium so that the force between
them is F/5? [Shanti Asiatic] Ans. √ 5d [1 mark]
13. The electrostatic force on a small sphere of charge 0.4 μC due to another small sphere of charge – 0.8 μC in
air is 0.2 N. (a) What is the distance between the two spheres? (b) What is the force on the second sphere
due to the first? [Textbook] Ans. (a) 0.12 m, (b) -0.2N [2 marks]

14. Four point charges Q, q, Q and q are placed at the corners of a square of side ‘a’ as shown in the figure 73.
Find the resultant electric force on a charge Q. [Board
2018] [2 marks]

15. Three point charges q, -4q and 2q are placed on the vertices of an equilateral triangle ABC of side ‘l’. Obtain
the expression for the magnitude of the resultant electric force acting on the charge q. [Board 2018]
[2 marks]
16. Two equal positive charges, each of 2C interact with a third positive charges of 3 C
situated as shown in figure 74. Find the magnitude and direction of the force
experienced by the charge of 3C. Ans. 3.456×10-3 N, 37 wrt to either force
[3 marks]

4th floor, Sigma excellence, opposite Falguni grah udhyog, Vastrapur, Ahmedabad, 9979203664, 079-40327045
56

17. Which among the curves shown in Fig. 75 cannot possibly represent electrostatic field lines?

[2 marks]

18. An electric dipole with


dipole moment 4 × 10-9 C m
is aligned at 30° with
4 -1
the direction of a uniform electric field of magnitude 5 × 10 NC . Calculate the magnitude of the torque
acting on the dipole. [Textbook] Ans.10-4Nm
[2 marks]
19. An electric dipole consists of charges +2e and -2e separated by 0.78 nm. It is in an electric field of strength
3.4×106 N/C. Calculate the magnitude of the torque on the dipole when the dipole moment is (a) parallel to
(b) perpendicular to, and (c) antiparallel to the electric field. Ans. (a) Zero (b) 8.5×10 -22Nm, (c) Zero
[2
marks]
20. A point charge of 2.0C is at the centre of a cubic Gaussian surface 9.0cm on edge. What is net electric flux
through the surface? Ans. 2.26105 Nm 2 C−1 [2 marks]

21. A hemispherical body of radius R is placed in a uniform electric field E. What is the flux linked with curved
surface area, if field is (a) parallel to the base, and (b) perpendicular to
base? Ans. (a) Zero, (b) EπR2 [2 marks]
22. Two large, thin metal plates are parallel and close to each other. On
their inner surface, the plates have surface charge densities of opposite
signs and of magnitude 17.010−22 C/m 2. What is Electric field: (a) in the
outer region of the first plate, (b) in the outer region of the second
plate, and (c) between the plates? Ans. (a) Zero, (b) Zero, (c) 1.9210−10 N/C [2 marks]
23. Figure 77 shows three point charges, + 2q, – q and + 3q. Two charges + 2q
and – q are enclosed within a surface ‘S’. What is the electric flux due to

4th floor, Sigma excellence, opposite Falguni grah udhyog, Vastrapur, Ahmedabad, 9979203664, 079-40327045
57

q
this configuration through the surface ‘S’? [Delhi 2010] Ans. φ =
ε0
[1 mark]
24. A point charge +10 μC is a distance 5 cm directly above the centre of a square of
side 10 cm, as shown in Fig 78. What is the magnitude of the electric flux
q
through the square?[Textbook] [Hint: Flux = ¿ Ans.
6 ε0
1.88×105Nm2/C [2 marks]

Physical Quantities, SI Units and Dimensions


Physical Quantity SI Unit Scalar/ Dimensions
Vector
Charge (Q) Coulomb (C) Scalar [AT]
Electrostatic Force (F) Newton (N) Vector [MLT -2]
Electric Field (E) N/C or V/m Vector [MLT -3A-1]
Permittivity of medium (ε) C2/Nm2 or Farad/m or C/Vm Scalar [M-1L-3T4A]
Dielectric Constant (K) Nm2/C2 or V/m Scalar [ML3T -4A-1]
Dipole moment (p) Cm Vector [LAT]

4th floor, Sigma excellence, opposite Falguni grah udhyog, Vastrapur, Ahmedabad, 9979203664, 079-40327045
58

Torque (τ) Nm Vector [ML2T -2]


Linear Charge Density (λ) C/m Scalar [L-1AT]
Surface Charge Density (σ) C/m2 Scalar [L-2AT]
Volume Charge Density (ρ) C/m3 Scalar [L-3AT]
Electric Flux (φ) Nm2/C or Vm Scalar [ML3T -3A-1]
List of Formulas

Quantity Formula Quantity Formula


Electric Force (or k q1 q2 Torque due to dipole τ = pE sinθ or τ⃗ = ⃗p × ⃗
E
Coulomb’s force) F=
r2
Electric Field due to kq Electric Flux E . ⃗S = ES cosθ
φ=⃗
point charge E= 2
r
Relation b/w F = q⃗
⃗ E Gauss Law ❑
q
E.⃗
ΦE = ∮ ⃗ dS =
electric force and
S ε0
field
Dipole Moment ⃗p = q × 2a Electric field due to 2k λ
E ∨¿=
|⃗
infinitely long charged r
wire
Electric field due to 2 kr ⃗p Electric field due to σ
E=
⃗ 2 2 & E¿
dipole at axial point 2
(r −a ) uniformly charge infinite 2 ε0
2 k ⃗p plane sheet
for r>>2a, ⃗E= 3
r

Electric field due to k ⃗p Electric field due to kq


E=
⃗ 2 2 3 /2 & E| ¿
(i) Outside the shell: |⃗ ¿
dipole at equatorial (r +a ) uniformly charged thin r2
point −k ⃗p spherical shell σR 2
E= 3
for r>>2a, ⃗
r ϵ0 r2
kq σ
E| =
(ii) On the shell: |⃗ =
R 2 ϵ0
(iii) Inside the shell: E=0
Derivation List
1) Vector form of Coulomb’s Law.
2) Relation between Electric field and r due to point charge using Electric Field Lines.
3) Electric field due to electric dipole at axial and equatorial points.
4) Torque and net force on electric dipole in uniform electric field.
5) Deduction of Coulomb’s law from Gauss’s law.
6) Applications of Gauss’s Law:
a. Electric field due to infinite line charge at distance ‘r’.
b. Electric field due to infinite plane sheet
c. Electric field due to two infinite plane sheets of opposite charge densities.
d. Electric field due to spherical shell of radius ‘R’(a) outside the shell, (b) inside the shell, (c) on the shell

Do it yourself

4th floor, Sigma excellence, opposite Falguni grah udhyog, Vastrapur, Ahmedabad, 9979203664, 079-40327045
59

1. Gold Leaf Electroscope:


Principle: Like charges repel each other.
Construction: It consists of a metallic disc outside
a glass chamber. This metallic disc is attached to a
metal stem in glass chamber which is further
attached to a metal plate and a thin gold leaf
which are all capable of conducting electricity.
Working:
(i) When a charged object is touched to the disc,
gold leaf acquires the same charge as the
charged object by conduction and repels
metal plate and diverges. The degree of divergence is an indicator of the amount of charge.
(ii) Now, if an object with opposite charge in brought near the metal disc, then the metal plate and gold strip
will hang straight down because the charge on them has been neutralised. After neutralisation,
electroscope becomes discharged.
Uses: 1) finding whether the given object is charged or not. 2) To find extent of charging(indicated by degree of
divergence of leafs). 3) To find type of unknown charge on object using an object carrying known charged .
Question:
What will happen to leaves of an electroscope if a positively charged body is brought near (a) uncharged
electroscope (b) positively charged electroscope and (c) negatively charged electroscope? Explain.
[DPS] [2
marks]
Sol: (a) Leaves remain unaffected as there is no charge on leaves of electroscope.
(b) Divergence of leaves increases as positive charge from body transfers to metal plate and hence repulsion
between leaves and metal takes place.
(c) Divergence of leaves decreases as negative charge from body transfers to metal plate and hence attraction
between leaves and metal takes place.
2. Quarks (Just For Knowledge): Protons, neutrons and electrons are believed to be made up of more
2
fundamental particles called quarks. Quarks are of two types: up quark (U) possessing + e charge and down
3
1
quark (D) possessing - e charge. The independent existence of quarks is not detected so far. Proton
3
represented by UUD and Neutron by UDD.
3. Deriving relation between E & r using concept of field lines

Let us consider two points P1 & P2 at distances r1 & r2 respectively from a point charge +q.
ΔS 1
At P1, ΔΩ= →①, where ΔS 1 is area at P1 , subtending the ΔΩ at O.
r 21
ΔS 2
Similarly at P2, ΔΩ = →②
r 22
Let no. of field lines cutting area S 1 & S2 be n, then according to definition,
n n
E1= & E2 = , from equation ① & ②,
∆ S1 ∆ S2
n n
E1= 2 & E2 =
∆ Ω r1 ∆ Ω r 22
2
E1 4th rfloor,
2 1
Hence, = 2 SigmaE excellence,
. opposite Falguni grah udhyog, Vastrapur, Ahmedabad, 9979203664, 079-40327045
E2 r r2
60

4. Deduction of Coulomb’s Law from Gauss’s Law: [2 marks]

E at each pt. of surface of


Consider a unit positive charge at O. Imagine a sphere of radius r with centre O. ⃗
⃗ ⃗ dS hence θ=00 .
sphere is same & directed radially outwards. For a small area vector dS , E is parallel to ⃗
According to Gauss’s law,

q
∮ ⃗E . ⃗
dS =
ε
S 0

q
E ∮⃗
⃗ dS cos 00 =
S ε0
q
E.4πr 2 =
ε0
kq
E ¿ 2 →①
r

4th floor, Sigma excellence, opposite Falguni grah udhyog, Vastrapur, Ahmedabad, 9979203664, 079-40327045

You might also like